Sei sulla pagina 1di 125

8.2!

1.A 47-year-old man is extricated from an automobile after a motor


vehicle accident. He is hypotensive with a systolic blood pressure
of 80. The patient has a steering wheel bruise on the anterior
chest. His electrocardiogram (ECG) shows some premature
ventricular complexes, and his cardiac isoenzymes are elevated.
Which of the following is the best next test for evaluation for a blunt
cardiac injury?
a. Measurement of serial creatinine phosphokinase and creatinine
kinase (including the myocardial band) levels
b. Thallium stress test
c. Echocardiography
d. Single photon emission computed tomography (SPECT)
e. Multiple acquisition scans (MUGA)
Echocardiography provides a sensitive assessment of ventricular wall motion
and ejection fraction after blunt chest trauma and is indicated in
hemodynamically unstable patients,
!
2. A 45-year-old man presents after a high-speed motor vehicle

collision. He has a seatbelt sign across his neck and chest with an
ecchymosis over his left neck. He is hemodynamically stable and
neurologically intact. A CT angiogram shows a left carotid
dissection. In the absence of other significant injuries, what is the
next step in his management?
a. Antiplatelet therapy
b. Systemic anticoagulation with heparin
c. Neck exploration and left carotid artery repair
d. Neck exploration and left intra-extracranial bypass
e. Angiography and left carotid artery stenting
in a blunt carotid artery injury :
One of the greatest concerns after BCVIs is stroke secondary to
thromboembolism developing from the disrupted vessel wall. , so if there are
no contraindication , must give systemic anticoagulation

4. A 27-year-old man has bulky retroperitoneal adenopathy after

radical orchiectomy for a mixed germ cell tumor. His chest x-ray is
normal. Serum beta- human chorionic gonadotropin (-hCG) and
alpha-fetoprotein (AFP) are markedly elevated. Liver enzymes are
slightly elevated, and the patient relates a history of ethanol
excess. He receives three cycles of chemotherapy. Restaging
reveals a 3- cm retroperitoneal mass, a normal chest x-ray, and
normal serum -hCG. However, the serum AFP is 20 IU/mL
(normal = 0 to 9 IU/mL). What is the next step in the management
of this patient?
a. Computed tomography (CT)guided needle biopsy
b. External-beam radiotherapy
c. Retroperitoneal lymph node dissection
d. Salvage chemotherapy
e. Serial markers and CT scans
This patient presents with a residual bulky mass after three courses of
platinum-based chemotherapy. Although the chest x-ray and -hCG are
normal, the serum AFP remains slightly elevated. AFP production is usually
attributed to yolk sac elements in a mixed germ cell tumor. It is also seen with
a number of other conditions, such as hepatocellular carcinomas and benign
hepatic disease, including alcohol hepatitis, as is probable in this case.
Patients with persistent marker elevations after chemotherapy are usually
considered very likely to harbor residual carcinoma and probably best
managed by further chemotherapy. However, the AFP elevation seen in this
case is more likely due to benign liver disease. Consequently, this patient
would be best managed by retroperitoneal lymph node dissection instead.
The most likely finding at retroperitoneal lymph node dissection would be
either fibrosis or residual teratoma. CT scandirected percutaneous needle
biopsy would have considerable sampling error, and external-beam
radiotherapy has no efficacy, particularly in the management of teratoma

5. A 57-year-old man is brought to the emergency department

complaining of dyspnea and chest pain. He also admits to a 20-lb


weight loss. He complains of fevers, chills, and night sweats.
Physical examination reveals supraclavicular adenopathy. Chest
examination reveals distant heart sounds. Laboratory studies
reveal a white blood cell count of 170,000/mL. Chest x-ray and
echocardiography reveal a pericardial effusion. What is the most
likely explanation of these findings?
a. Atrial myxoma
b. Atrial fibrillation
c. Lymphoma
d. Metastatic colorectal carcinoma
e. Pericarditis
6. A 25-year-old woman arrives in the ER following an automobile
accident. She is acutely dyspneic with a respiratory rate of 60
breaths per minute. Breath sounds are markedly diminished on the
right side. Which of the following is the best first step in the
management of this patient?
a. Take a chest x-ray.
b. Draw arterial blood for blood-gas determination.
c. Decompress the right pleural space.
d. Perform pericardiocentesis.
e. Administer intravenous fluids.
7. A 19-year-old woman began breast-feeding for the first time. At
first, it was difficult for her infant to feed. Now, her breasts are red,
warm, and sore. She has continued to breast-feed, despite the
pain; however, she has recently begun to use a breast pump
instead of breast-feeding. She is begun on a course of oral
antibiotics. What condition is this patient at risk of developing?
a. Breast abscess
b. Fibrocystic disease
c. Inflammatory breast cancer
d. Prolactinoma
e. Tuberculosis

8. Surgery is indicated in the initial management of lung cancer in


the presence of which of the following?
(A) Hypercalcemia
(B) Vocal cord paralysis
(C) Superior vena cava syndrome
(D) Small-cell anaplastic carcinoma
(E) Chest wall and anterior abdominal wall metastasis
Hypercalcemia is attributed to the secretion
of parahormone from a localized squamous
cell carcinoma (paraneoplastic effect); as
such, improvement may be seen after surgical
resection.
9. Which of the following tricyclic anti depressants closely

resembles an SSRI in its action mechanism?


a. amitriptyline
b. nortriptyline
c. imipramine
d. clomipramine
e. doxepine
10. A 28-year-old medical student seeks your attention because of
a testicular mass. Biopsy is consistent with pure seminoma. There
is no evidence of enlarged retroperitoneal lymph nodes on CT
scan. Which of the following is the best treatment strategy for this
patient?
a. Orchiectomy alone
b. Orchiectomy followed by chemotherapy
c. Orchiectomy with retroperitoneal lymph node dissection
d. Orchiectomy with retroperitoneal lymph node dissection followed
by external beam radiation
e. Orchiectomy followed by external beam radiation to the
retroperitoneal lymph nodes
Early-stage seminomas should be treated with orchiectomy and external
beam radiation. Seminoma is extremely radiosensitive; therefore
retroperitoneal lymph node dissection is unnecessary.

11. Dysthymic disorder:

a. Usually begins in adulthood


b. Does not respond to antidepressants
c. Presents with symptoms of lack of say in life and preoccupation
with inadequacy
d. Usually limited to one or two episodese. Characteristically
marked by psychomotor agitation or retardation
12. Twelve year old child is eing followed in clinic for asthma, his
treatment is inhale but he is still coughing you are considering
wether to increase the steroid dosing, which of the following will
assisst with desicion making:
a. pulmonary function test
b. chest x-ray
c. seum IgE levels
d. CBC wit differential
e. skin test
13. A 53-year-old woman presents with weight loss and a
persistent rash to her lower abdomen and perineum. She is
diagnosed with necrolytic migrating erythema and additional
workup demonstrates diabetes mellitus, anemia and a large mass
in the tail of the pancreas. Which of the following is the most likely
diagnosis?
a. Verner-Morrison syndrome (VIPoma)
b. Glucagonoma
c. Somatostatinoma
d. Insulinoma
e. Gastrinoma

14. A 53-year-old woman presents with complaints of weakness,


anorexia, malaise, constipation, and back pain. While being
evaluated, she becomes somewhat lethargic. Laboratory studies
include a normal chest x-ray, serum albumin 3.2 mg/dL, serum
calcium 14 mg/dL, serum phosphorus 2.6 mg/dL, serum chloride
108 mg/dL, blood urea nitrogen (BUN) 32 mg/dL, and creatinine
2.0 mg/dL. Which of the following is the most appropriate initial
management?
a. Intravenous normal saline infusion
b. Administration of thiazide diuretics
c. Administration of intravenous phosphorus
d. Use of mithramycin
e. Neck exploration and parathyroidectomy
15. A 33-year-old pregnant woman notices a persistent, painless
lump in the left breast. On examination the left breast has a single
mobile mass without evidence of skin changes or
lymphadenopathy in the neck or axilla. An ultrasound
demonstrates a solid, 1-cm mass in the upper outer quadrant of
the breast. A core-needle biopsy shows invasive ductal carcinoma.
The patient is in her first trimester of pregnancy. Which of the
following is the most appropriate management of this patient?
a. Termination of the pregnancy followed by modified radical
mastectomy
b. Immediate administration of chemotherapy followed by modified
radical mastectomy after delivery of the baby
c. Administration of radiation in the third trimester followed by
modified radical mastectomy after delivery of the baby
d. Total mastectomy with sentinel lymph node biopsy
e. Modified radical mastectomy

16. A 40-year-old woman presents with a rash involving the nippleareola complex for the last month with associated itching. On
physical examination there is crusting and ulceration of the nipple
with surrounding erythema involving the areola and surrounding
skin, no palpable breast masses, and no cervical or axillary
lymphadenopathy. Which of the following is the most appropriate
next step in the management of this patient?
a. Reexamine the patient in 1 month
b. Corticosteroid cream to the affected area
c. Administration of oral antibiotics
d. Mammogram and biopsy of the affected area
e. Modified radical mastectomy
17. A 29-year-old woman with a history of difficulty becoming
pregnant presents to her primary care physician and is diagnosed
with Grave disease on iodine uptake scan; her thyrotropin (TSH)
level is markedly suppressed and her free thyroxine (T4) level is
elevated. She desires to conceive as soon as possible and elects
to undergo thyroidectomy. After she is rendered euthyroid with
medications preoperatively, which of the following management
strategies should also be employed to reduce the risk of
developing thyroid storm in the operating room?

a. Drops of Lugol iodine solution daily beginning 10 days


preoperatively.
b. Preoperative treatment with phenoxybenzamine for 3 weeks.
c. Preoperative treatment with propranolol for 1 week.
d. Twenty-four hours of corticosteroids preoperatively.
e. No other preoperative medication is required.

18. A 64-year-old man complains of pain in the lower chest. ACT


scan confirms the presence of a tumor of the lung at T10 level to
the left of the
midline and invading the surrounding left lung base. Because of
the structure most likely involved and penetrating the diaphragm at
this
level, what could be associated?
(A) Hoarseness
(B) Latissimus dorsi palsy
(C) Budd-Chiari syndrome (hepatic venous outlet obstruction)
(D) Dysphagia
(E) Tracheobronchial fistula
19. A 52-year-old patient presents for her annual examination. She
denies any problems other than that she has not had an annual
examination in over 5 years due to cost. During the breast
examination she is noted to have dimpling of the skin of the right
breast with raising of the arms. What possibility should this sign
signify?
(A) pregnancy
(B) weight gain
(C) aging
(D) fibrocystic disease
(E) carcinoma
The fascia of the superficial chest muscles becomes condensed into bands called
Coopers ligaments. These bands run from the base of the breast to the skin to
provide support for the breast. Distortion of these bands by a tumor may cause
dimpling of the skin overlying the breast. Such dimpling is considered a sign of
malignancy.

In chest surgery, which is true regarding a thoracoabdominal


incision?
(A) It should be used for most abdominal and thoracic procedures.
(B) It enters the third to fifth intercostal space.
(C) It causes less postoperative pain.
(D) It allows division of the costal margin and the diaphragm.
(E) It causes severe denervation of the anterior abdominal wall.
20.

21. Squamous cell carcinoma of the lip is least likely to develop in


which of the following?
(A) Scandinavian fisherman
(B) Redheaded pornographic actress with a gorgeous year-round
tan
(C) Man from Lohatchie, AL, who smokes a clay pipe
(D) Brunette secretary who constantly drinks tea
(E) Mentally defective man who smokes 40 cigarettes a day and
keeps the butt in his mouth
22. Prior to initiating estrogen replacement therapy (ERT), your
patient is counseled regarding the long- term risks of estrogen
deficiency associated with menopause. A major concern is
osteoporosis. With osteoporosis, the accelerated bone loss
occurring in the first 1 to 8 years after menopause is associated
with which of the following?
(A) an elevation in circulating parathyroid hormone levels
(B) increased urinary loss of phosphorus and hydroxyproline
(C) no influence on trabecular bone
(D) primarily with affects on cortical bone
(E) X-ray can diagnose early osteoporosis (osteopenia)
23. which of the following children should be admitted for urinary
tract infection?
a. a 1 year child that also passed diarrhea X3 today
b. a 6 year girl complaining of flank pain
c. a 5 year girl with a history of uti at the age of 1 year
d. a 3 week old infant with fever 38.5c
e. a 4 year girl with fever of 39c, with burning sensation and
urinary urgency
A doesn't have any uti symptom
B flank pain without fever is , most propably not a UTI
C definitely needs a work up to R/O reflux but doesn't need to be admitted
E is the typical lower UTI that could be treated as an outpatient

24. A 34-year-old woman complains that she is gaining weight


since she started taking chlorpromazine for chronic
undifferentiated schizophrenia. A complete review of systems and
history shows evidence of breast enlargement and lactation.
However, a serum pregnancy test is negative. Which of the
following antipsychotics would be a good choice for a treatment
change?
A Fluphenazine
B Haloperidol
C Paliperidone
D Perphenazine
E Aripiprazole
24. A disheveled 27-year-old man presents with bizarre
movements, agitated behavior, and rapid and unintelligible speech.
The patient has had signs of psychosis for much of the past few
months, and the family reports that the patient twice spent a month
in state psychiatric hospitals. Which of the following schizophrenic
subtypes can most appropriately be applied to this patient?
A Chronic undifferentiated
B Catatonic
C Residual
D Paranoid
25. A 29-year-old man who lives in a group home and takes
perphenazine on a daily basis develops a high fever and muscle
rigidity. The man undergoes examination and testing at the local
emergency department, where a creatine phosphokinase value
comes back as 10,000. Which of the following ther- apeutic steps
should be taken next?
A Send the patient home on an atypical antipsychotic.
B Send the patient home with instructions to take acetaminophen
every 4 hours.
C Admit the patient to a state hospital
D Admit the patient to an intensive care unit.
E Send the patient home on benztropine (2 mg tid).
Neuroleptic malignant syndrome is potentially life threatening and requires

admission to an intensive care unit for support of respiratory, renal, and


cardiovascular functioning.

26.True statements about the lengths of time drugs of abuse can


be detected in urine include
A. alcohol for 7 to 12 hours
B. benzodiazepine for 2 to 3 weeks
C. cocaine for 1 to 2 weeks
D. marijuana for 24 to 48 hours
E. morphine for 8 days

27. An 8-year-old girl presents for a pediatric visit because her


teacher has raised concerns about her lack of talking at school.
Her mother tells you that her daughter talks all the time at home
and has many interests and activities. She also describes her
daughter as having achieved all developmental milestones at
appropriate ages. When she is at school, however, she does not
speak spontaneously and will only speak when spoken to directly
by her teacher. She is having difficulty socially, as she does not
reciprocate social interactions with her peers. When she is at
home, she plays actively with her two sisters and talks normally.
What do you tell the mother her daughter suffers from?
a. Major depression
b. Posttraumatic stress disorder
c. Autism
d. Selective mutism
e. Abulia
28. A good test for recent memory is to ask patients
A. to subtract 7 from 100!
B. their date of birth
!C. how many siblings they have!
D. what they had to eat for their last meal
E. who is the president of the United States

29. A 46 old female who is 6 week postpartum complains of feeling


low in her mood and having crying spells, lack of motivation and
feeling of hopelessness for the past 4 week she is diagnosed with
postpartum depression.
Which of the following is true about postpartum depression?
a. Risk of infanticide is minimal
b. Selective serotonin reuptake inhibitors (SSRIs) are
contraindicated
c. Risk of depression during subsequent pregnancies is increased
d. Suicide risk is low
e. Hospital admission is rarely required
30.!!Polysomnography!(sleep!EEG)!abnormalities!include!!
A.!an!increase!in!REM!sleep!in!dementia!!
B.!an!increased!sleep!latency!in!schizophrenia!!
C.!a!decrease!in!the!amount!of!REM!sleep!in!major!depressive!
disorder!!
D.!a!lengthened!REM!latency!in!major!depressive!disorder!!
E.!none!of!the!above!!
!
!
!
!
!
!
!
!
!

!
!
!
31. A 19-year-old man is brought to the physician by his parents
after he called them from college, terrified that the Mafia was after
him. He reports that he has eaten nothing for the past 6 weeks
other than canned beans because they are into everythingI
cant be too careful. He is convinced that the Mafia has put
cameras in his dormitory room and that they are watching his
every move. He occasionally hears the voices of two men talking
about him when no one is around. His roommate states that for the
past 2 months the patient has been increasingly withdrawn and
suspicious. Which of the following is the most likely diagnosis?
a. Delusional disorder
b. Schizoaffective disorder
c. Schizophreniform disorder
d. Schizophrenia
e. Phencyclidine (PCP) intoxication
32. Amnestic disorders :
A. are secondary syndromes caused by primary etiologies.
B. are most often caused by nutritional deficiencies related to
chronic alcohol dependence.
C. may be diagnosed in the context of delirium.
D. may be diagnosed in the context of dementia.E. none of the
above!

33. A 22-year-old man is brought to the emergency room after he


became exceedingly anxious in his college dormitory room, stating
that he was sure the college administration was sending a hit
squad to kill him. He also notes that he can see visions of men
dressed in black who are carrying guns and stalking him. His
thought process is relatively intact, without thought blocking or
loose associations. His urine toxicology screen is positive for one
of the following drugs. Which drug is the most likely cause of these
symptoms?
a. Barbiturates
b. Heroin
c. Benzodiazepines
d. Amphetamines
e. MDMA (Ecstasy)
34. A 52-year-old man is seen by a psychiatrist in the emergency
room because he is complaining about hearing and seeing
miniature people who tell him to kill everyone in sight. He states
that these symptoms developed suddenly during the past 48
hours, but that he has had them on and off for years. He states
that he has never previously sought treatment for the symptoms,
but that this episode is particularly bad. He denies the use of any
illicit substances. The patient is alert and oriented to person, place,
and time. His mental status examination is normal except for his
auditory and visual hallucinations. His thought process is normal.
His drug toxicology screen is positive for marijuana but no other
substances. He is quite insistent that he needs to be put away in
the hospital for the symptoms he is experiencing. Which of the
following is the most likely diagnosis?
a. Substance-induced psychosis
b. Schizophrenia
c. Schizoaffective disorder
d. Schizophreniform disorder
e. Malingering

35. A 22-year-old college student calls his psychiatrist because for the
past week, after cramming hard for finals, his thoughts have been racing
and he is irritable. The psychiatrist notes that the patients speech is
pressured as well. The patient has been stable for the past 6 months on
500 mg of valproate twice a day, and a blood level is in the therapeutic
range. Which of the following is the most appropriate first step in the
management of this patients symptoms?
a. Hospitalize the patient
.!b. Increase the valproate by 500 mg/day
.!c. Prescribe clonazepam 1 mg qhs.!
d. Start haloperidol 5 mg qd.!
e. Tell the patient to begin psychotherapy one time per week.
Sleep deprivation has an antidepressant effect in depressed patients and may
trigger a manic episode in bipolar patients. The patient is not ill enough to
require hospitalization. The use of a long-acting benzodiazepine will allow the
patient to return to a normal sleep pattern and generally will abort the manic
episode.

Bipolar I disorder: At least one manic episode is necessary to


make the diagnosis;[74] depressive episodes are common in
bipolar disorder I, but are unnecessary for the diagnosis.
Bipolar II disorder: No manic episodes, but one or more hypomanic
episodes and one or more major depressive episode.Hypomanic
episodes do not go to the full extremes of mania (i.e., do not
usually cause severe social or occupational impairment, and are
without psychosis), and this can make bipolar II more difficult to
diagnose, since the hypomanic episodes may simply appear as a
period of successful high productivity and is reported less
frequently than a distressing, crippling depression.

36.All of the following statements are factors with an increased risk


of schizophrenia except
A. having a schizophrenic family member
B. having a history of temporal lobe epilepsy
C. having low levels of monoamine oxidase, type B, in blood
platelets
D. having previously attempted suicide
E. having a deviant course of personality maturation and
development
37. Which of the following are not associated with atypical depression:
A. increase appetite with unintentional weight gain
B. increase desire to sleep, usually more than 10 hours a day
C. heavy, leaden feeling in your arms and legs that lasts an hour or
more in a day
D. sensitivity to rejection or criticism, with affects your
relationships
E. depression that does not react or lifts when something good
occurs
38. Mild, nonpsychotic depression with predominant anxiety is
called :
A. Endogenomorphic depression
B. Bipolar disorder
C. Chronic depression
D. Dysthymia
E. Anxiety disorder
39. A 27-year-old patient has been diagnosed with bipolar
disorder. Before starting this patient on lithium for mood
stabilization, which of the following laboratory tests should be
obtained?
A. Thyroid function tests, creatinine, pregnancy test
B. Thyroid function tests, creatinine, liver function tests
C. Thyroid function tests, creatinine, complete blood count

D. Thyroid function tests, liver function tests, pregnancy test


E. Thyroid function tests, complete blood count, pregnancy test

40. A 5-month-old boy comes to you for follow-up of his poor


weight gain. You note that he is a bright and cheerful child, but with
persistently poor weight gain and linear growth. Additionally, you
note a mild degree of clubbing of his fingers. When getting ready
for today's trip to your clinic, the child became agitated and started
to turn blue. His mother strapped him into his car seat to hurry to
your office and noted that his color improved and he seemed
comfortable again. Which of the following diagnoses best explains
his particular symptoms?
A. CF
B. Tetralogy of Fallot
C. Hypoplastic left heart syndrome
D. Hirschsprung disease
E. breath holding spells
Tetralogy of Fallot is a complex congenital heart disease that includes four
distinct abnor- malities: VSD, pulmonary stenosis, overriding aorta, and
RV hypertrophy. The increase in pul- monary vascular resistance due to
crying or agi- tation increases the right-to-left shunt of blood flow in the
heart and causes hypoxemia and cyanosis. This is known as a "Tet spell."
Sitting with the knees up toward the chest, as in a car seat or in a squatting
position, increases sys- temic vascular resistance, and reduces the degree
of right-to-left shunting, improving oxy- genation.
-WHY NOT E ? Breath-holding spells are typically provoked by an upsetting
circumstance for the child. They usually present from age 2 to 5 years, and are
rare before 6 months of age. They would not be expected to be accompanied by
poor weight gain or clubbing on exam

41.. A 6 month old boy is found to have very low levels of IgG, IgM,

and IgA. Which of the following organisms is mostlikely to cause


problems in this patient?
(A) enterovirus
(B) herpes virus
(C) Shigella
(D) Escherichia coli

(E) Mycobacterium tuberculosis

42. A 38-year-old woman with bipolar disorder has been stable on


lithium for the past 2 years. She comes to her psychiatrists office in tears
after a 2-week history of a depressed mood, poor concentration, loss of
appetite, and passive suicidal ideation. Which of the following is the most
appropriate next step in the management of this patient?
a. Start the patient on a second mood stabilizer.!
b. Start the patient on a long-acting benzodiazepine.
c. Stop the lithium and start an antidepressant.
!d. Start an antidepressant and continue the lithium.
e. Stop the lithium and start an antipsychotic.

43. A 19-year-old woman comes to the psychiatrist for a history of anger


and irritability, which occurs once monthly on average. During this time
the patient also reports feeling anxious and about to explode, which
alternates rapidly with crying spells and angry outbursts. The patient
notes during this time she cant concentrate and sleeps much more than
she usually needs to do. During the several days these symptoms last, the
patient must skip most of her classes because she cannot function. Which
of the following is the most likely diagnosis?
a. Adjustment disorder with depressed mood
b. Major depression!
c. Premenstrual dysphoric disorder!
d. Dysthymic disorder
e. Depressive personality disorder

44.At what gestational age should laboratory testing for gestational


diabetes (50 Gr glucose challange test) be performed?
A. 10-16 weeks
B. 20-24 weeks
C. 24-28 weeks
D. 33-36 weeks
45. Which of the following infectious agents have been associated
with the onset of obsessive-compulsive disorder (OCD)?
A. Streptococcal infection
B. Borrelia Burgdorferi
C. Retrovirus
D. Influenza virus
E. Varicella zoster virus
46. Which of these statements regarding the prognosis of
dissociative identity disorder is incorrect?
A. Recovery is generally complete.
B. The earlier the onset of dissociative identity disorder, the poorer
the prognosis is.
C. The level of impairment is determined by the number and types
of various personalities.
D. Individual personalities may have their own separate mental
disorders.
E. One or more of the personalities may function relatively well.
47. A gradual, smooth deceleration of the fetal heart rate that
follows the peak of a contraction describes which of the
following deceleration types?
A. Late
B. Early
C. Variable
D. Prolonged

48. A 21-year-old man comes to the physician because of excessive


sleepiness. He states that for the past 4 months he becomes so sleepy that
he must sleep, even when he is in the middle of an important meeting.
These episodes occur daily and the patient must sleep for 10 to 20
minutes at each episode. The patient also says that on several occasions
he has had a sudden loss of muscle tone during which his knees become
weak and he drops to the floor. He remains conscious during these
episodes. He denies any substance abuse or medical problems. Which of
the following is the most appropriate treatment to be started?
a. Benzphetamine
b. Valproic acid
c. Lithium!d.
d.Modafinil
e. Nasal continuous positive airway pressure
50. Which of the following is a contraindication for ECT?
a. Space-occupying lesion in the brain
b. Pregnancy!
c. Hypertension!
d. Seizure disorder
e. Status postmyocardial infarction 6 months earlier

51. What is not true about ovarian stimulation?


a. Can be triggered by recombinant or urinary gonadotropins
b. There is indication for serum estradiol test and US for follicular
monitoring
c. HCG give at least one follicle at size 14mm
d. GnRH agonist and antagonist given to prevent
e. Ovulation occur 36hr after HCG administration
C- it's gives at least three follicule

52. A 14-year-old girl has a chronic cough with copious

expectoration. A biopsy of the respiratory mucosa shows ciliated


epithelium devoid of dynein arms. Which of the following conditions
is most likely to occur in later life?
(A) abnormal vaginal bleeding
(B) chronic diarrhea
(C) infertility
(D) pelvic pain(
E) urinary incontinence
(C) Immobile cilia syndrome, or Kartagener syndrome, refers to the congenital
absence of dynein arms in ciliated epithelium. Abnormalities of microtubular
structure may coexist. Patients with Kartagener syndrome have chronic cough,
sinusitis, bronchiectasis, and airway obstruction. The diagnosis is usually made by
microscopic examination of the respiratory mucosa. Immobile cilia may coexist in the
fallopian tube epithelium and may increase the risk of infertility in some individuals.

53. Which of the following is correct regarding testosterone?


a. 50% of the testosterone in produced by the ovary
b. 80% is binded of SHBG and 19% is binded to albumin
c. The testosterone that is binded to SHBG is the active
testosterone
d. 25% of the testosterone is peripheral formed
e. Circulating testosterone level in mature woman are about
100ng/dl
54. Which of the following is not an IVF complication?
a. Ovarian over stimulation
b. Fallopin tube pregnancies
c. Multifetal pregnancies
d. Heterotopic pregnancies
e. Ovarian cancer
54. A pregnant woman with HIV was admitted to hospital which of
the following regimen effect pneumoscystisis carnii
pneumonia:
A. AMPICILIN
B. ERYTHROMYCIN
C. COTRIMOXAZOLE
D. AZITHRHROMYCIN
E. VANCOMYCIN

55.!What!is!the!most!common!complication!of!Para!cervical!block?!
a.!Maternal!hypotension!!
b.!CNS!toxicity!!
c.!Fetal!bradycardia!!
d.!Bleeding!
e.!arrest!of!dilation!!
!

!
!
10.10!
!
1. A 74-year-old man is brought to the hospital because of urinary

retention. His past medical history includes hypertension, and


benign prostatic hypertrophy. A Foley catheter is inserted to relieve
the obstruction, and 1500 cc of urine is emptied from his bladder.
Over the next few hours, he has 200cc/h of urine output. Which of
the following urine electrolyte values is most likely in
keeping with his diagnosis of post obstructive diuresis?
(A) high sodium
(B) low potassium
(C) high specific gravity
(D) low pH
(E) osmolality >500 mOsm/kg

2. All of the following statements regarding pancreatic cancer are


true EXCEPT:
A. Alcohol consumption is not a risk factor for pancreatic cancer.
B. Cigarette smoking is a risk factor for pancreatic cancer.
C. Despite accounting for fewer than 5% of malignan- cies
diagnosed in the United States, pancreatic cancer
is the fourth leading cause of cancer death.
D. If detected early, the 5-year survival is up to 20%.
E. The 5-year survival rates for pancreatic cancer haveimproved
substantially in the past decade.

3. On screening, a 6 yr old adopted child has the following


biochemical and serologic findings:
ALT, 335 IU/L; positive HBsAg; positive HBcAb; and positive
HBeAg. Which of the following statements is true about interferon
therapy?
A.A realistic goal of therapy is to reduce the risk of the
development of hepatocellular carcinoma
B.A realistic goal of therapy is to induce hepatitis B surface
antibodies
C.Therapy is not approved and is not indicatedD.Duration of
therapy is typically 4 wkE.Therapy is typically not associated with
significant side effects

4. A 16 yr old girl with a past history of hypothyroidism, which


developed at age 10 yr, now manifests fever, anorexia,
amenorrhea, and jaundice of 4 months' duration. The direct
bilirubin level is 6 mg/dL, and the total bilirubin value is 11 mg/dL.
Results of the tests for hepatitis A, B, C, D, and E are negative,
and the serum IgG level is 16.5 g/L. The most likely diagnosis is:
A.Mononucleosis
B.Autoimmune hepatitis
C.1-Antitrypsin deficiency
D.Hepatoblastoma
E.Wilson disease

5.A 62-year-oldman presents to the emergency department


complaining of a droopy right eye and blurred vision for the past
day. The symptoms started abruptly, and he denies any
antecedent illness. For the past 4 months, he has been
complaining of increasing pain in his right arm and shoulder. His
primary care physician has treated him for shoulder bursitis without
relief. His medical history is significant for COPD and
hypertension. He smokes 1 pack of cigarettes daily. He has
chronic daily sputum pro- duction and has stable dyspnea on
exertion. On physical examination, he has right eye ptosis with
unequal pupils. On the right, his pupil is 2 mm and not reactive; on
the left, the pupil is 4 mm and reactive. However, his ocular
movements appear intact. His lung fields are clear to auscultation. On extremity examination, there is wasting of the intrinsic
muscles of the hand. Which of the following would be most likely to
explain the patients constellation of symptoms?
A. Enlarged mediastinal lymph nodes causing occlusion of the
superior vena cava
B. Metastases to the midbrain from small cell lung cancer
C. Paraneoplastic syndrome caused by antibodies to voltagegated calcium channels
D. Presence of a cervical rib on chest radiography
E. Right apical pleural thickening with a mass-like
density measuring 1 cm in thickness
pancoast tumor: horner + shoulder/arm pain!
6. What is the disease for which liver transplantation is most

commonly indicated? ( in children)


a.Alagille syndrome
b.Biliary atresia
c.Caroli disease
d.Hepatocellular carcinoma
e.Acetaminophen overdose

7. A 5 yr old presents in status asthmaticus, able to speak 2 words,


with scant wheezing,respirations of 40 breaths/min, and an oxygen
saturation of 95%. You provide 3 nebulized albuterol treatments
and the patient can now speak in sentences, has musical
wheezing throughout, and an oxygen saturation of 85%. The
decrease in the oxygen saturation is due to what phenomenon?
a.Poor pulse oximeter sensor reading
b.Nonresponse to albuterol due to poor medication delivery
c.Nonresponse to albuterol due to bronchomalacia
d.V/Q mismatch with "dead space ventilation"
e.intracardiac right-to-left shunting
8. All the following are suggestive of iron-deficiency anemia
EXCEPT:
A.Koilonychia
B.Pica
C.Decreased serum ferritin
D.Decreased total iron-binding capacity (TIBC)
E. Low reticulocyte response

When iron stores are depleted, serum iron begins


to fall. Total iron-binding capacity (TIBC) starts
to increase, reflecting the presence of circulating
unbound transferrin.
9. WHATS THE BEST METHOD TO DECREASE THE RISK OF RENAL FAILURE IN
A PATIENT WITH MYOGLUBINURIA?
IV HYDRATION TO MAINTAIN GOOD URINE OUTPUT

10. Aplastic anemia has been associated with all of the following
EXCEPT:
A. Carbamazepine therapy
B. Methimazole therapy
C. Nonsteroidal anti-inflammatory drugs
D. Parvovirus B19 infection
E. Seronegative hepatitis
Parvovirus B19 infection most commonly causes pure red blood cell
(RBC) aplasia, particularly in patients with chronic hemolytic states and
high RBC turnover (e.g., sickle cell anemia).

11. A 35-year-old woman is seeing a psychiatrist for treatment of her


major depression. After 4 weeks on fluoxetine at 40 mg/day, her
psychiatrist decides to try augmentation. Which of the following is the
most appropriate medication?
a. Lithium!
b. Sertraline!
c. An MAO inhibitor
d. Clonazepam
!e. Haloperidol

Lithium has been proven effective when added to an antidepressant in the


treatment of refractory depression. More than one mechanism of action is
probably involved, although lithiums ability to increase the presynaptic
release of serotonin is the best understood.

11. WITCH of the following has been showen to decrease wound


infection in a clean contaminated wound ( e.g nonperorated apendisitis )
- irrigation of the wound with salin solution proir to closuer )
12. Which of the following contraceptive methods shock syndrome
has been associated?
A) Oral contraceptive pill (OCP).
B) Progestin only pill.
C) Male condoms.
D) Vaginal diaphragma.
E) Intrauterine contraceptive device (IUCD).
13. Which of the following is a characteristic of female athletic
syndrome?
A) Menses response to progestron administration.
B) Pituitary hypertrophy.
C) No menses is response to estrogen and progestron
administration.
D) Hirsutism and acne.
E) Decreased bone mass.
14. WITCH TYPE OF MELANOMA HAS THE BEST OVERALL
PROGNOSIS ?
a.superficial spreading
b.nodular
c.letingo maligna
d.acral lentiginous
15. Marjolin's ulcer can arise in areas exposed to :
a.external beam radiation
b.thermal injury
c.pressure
d.lymphedema

16. During the treatment of septic shock, a 28-year-old male


remains hypotensive despite adequate volume replacement; PA
occlusion pressure is18 mm Hg. When dopamine is started ,
ventricular tachycardia develops and this is unresponsive to
lidocaine. The V-tach converts back to sinus rhythm once the
dopamine is stopped.At this point, which of the following
treatmentsare most appropriate for this hypotensive patient?(
A)Amrinone
(B)Dobutamine
(C)Epinephrine
(D)Phenylephrine
(E)Intra-aortic balloon pump
Dopamine activates b1-receptors and this
was probably the reason for the arrhythmia.
Amrinone will inhibit phosphodiesterase and
result in an increased cyclic AMPlevel, producing the same result as b-receptor stimulation. Dobutamine and epinephrine also
stimulate the b- receptors. The only choice
which stimulates only a-adrenergic receptors
is phenylephrine. Intra-aortic balloon pump
is invasive, therefore, less appropriate as a
17. B-blockers use in pregnancy in particular Atenolol is associated

with which of the following perinatal morbidities ?


A. preterm birth
B. neonatal hyperglycemia
C fetal grouth restriction
D. infuntile respiratory distress syndrome
E. neonatal polycythemia

18. You are asked to see a 70-year-old male admitted


to the ICU with anterior chest pain radiating to
the back described as a tearing sensation. The
pain reached maximum intensity within 30 minutes. The patient
has a history of hypertension
(noncompliant with medications). His BP in the ICU is 170/110 mm
Hg, HR = 110/min. Physicalexamination reveals a 2/6 diastolic
murmur andunequal femoral pulses. ACXR of this patientwas
normal and the CT chest, which wasobtained, is shown in Fig. 12.
Which of the following statements regarding his treatment
andprognosis are correct?
(A)The patient will require nitroprussideand beta blockade, but will
not requiresurgical intervention.
(B)The patient will require onlynitroprusside but will not
requiresurgical intervention.
(C)The patient will require nitroprusside andb-blockade prior to
emergency surgicalintervention.
(D)Neither nitroprusside nor b-blockade isrequired prior to surgical
intervention.
(E)Nitroprusside and b-blockade arerequired initially, but surgery
may bedone electively within 46 weeks.
19. Three teenaged high school students at the local school
develop meningitis. A classmate of theirs is concerned and comes
to the clinic for assessment and treatment. Which of the following
is the most appropriate next step in management?
(A) penicillin
(B) sulfonamides
(C) only reassurance and observation
(D) rifampin
(E) doxycycline

20. A 45-year-old premenopausal woman undergoes a left breast


lumpectomy for a 1.5-cm, lymph node positive, hormone sensitive
invasive breast cancer. She receives chemotherapy, radiotherapy,
and is on tamoxifen. Recommended follow-up after therapy
should always include:
(A) Blood tumor markers drawn every 36 months after treatment.
(B) Routine monitoring of liver function tests (LFTs) every 36
months after treatment.
(C) Yearly bone scans.
(D) Routine clinical examination every 36 months for the first 5
years after treatment as well as continued yearly mammography.
(E) Yearly breast MRI with gadolinium.

Follow-up after breast cancer treatment is very variable. There is no


consensus and no follow-up test has shown a survival advantage. Routine 36
month clinical examinations and yearly mammography should always be performed.

21. Which of the following is an indication for cholecystectomy in an asymptomatic


patient with an incidental finding of gallstones?
A.Any history of abdominal pain
B.Family history of complications of cholelithiasis
C.Porcelain gallbladder;
D.Frequent travel out of the country
22. You are caring for a 6-month-old baby with chronic cough. You order a sweat
chloride test and the results show the chloride is 104 mmol/L (normal is <40) on the
right arm and 108 mmol/L
on the left arm (with adequate volume of sweat at each site).
The next appropriate step is:
A. Repeat the sweat test for confirmation and refer to your local CF center
B. Refer to a geneticist for further evaluation
C. Obtain a stool elastase
D. Send a hair sample for analysis of mineral deficiency

E. Repeat the sweat test in 6 months to confirm the diagnosis

23. A 4-month-old male infant has been seen on three occasions with febrile
illnesses without localizing signs. He is breast fed, and his growth and
development have been normal. During weekdays he is in a daycare setting
while the mother works, and there are at least five other infants being cared
for in that daycare center. The mother is concerned about some sort of
immune deficiency that is making her infant susceptible to infections. Which
one of the following statements is true?
A. Hypogammaglobulinemia is a normal occurrence at 4 months
B. Infants are born with IgG, IgM, and IgA levels near normal adult values
C. Boys born with X-linked (Bruton) agammaglobulinemia usually develop
serious infections before 6 months of age
D. Infants with hypogammaglobulinemia will generally do poorly with ordinary
viral infections
E. Patients with hypogammaglobulinemia can usually handle infections with
encapsulated organisms normally
24. A mother has brought in her 8-year-old daughter because of the development of
breasts over the past year. They are now to the size that she is requiring a bra and is
being teased by the other children. Premature thelarche differs from true precocious
puberty in that premature thelarche is associated with which of the following? !
(A) axillary hair development
(B) isolated breast development
(C) pubic hair development
(D) spontaneous ovulations
(E) voice changes !

25. A 20-month-old male presents to your office with 5 days of fever to 104C that reduces
minimally with antipyretics. On exam he has a strawberry tongue, edema of the hands and
feet, and an erythematous maculopapular rash on his trunk. Which of the following is
required to make the diagnosis of Kawasaki disease?
A. Inguinal lymphadenopathy greater than 2 cm!
B. Nonexudative bulbar conjunctival injection!
C. Oral ulcerations
D. Mitral valve regurgitation on ECHO
E. Scrotal edema

26.A 41-year-old man complains of regurgitation of saliva and of undigested


food. An esophagram reveals a dilated esophagus and a birds-beak
deformity. Manometry shows a hypertensive lower esophageal sphincter with
failure to relax with deglutition. Which of the following is the safest and most
effective treatment of this condition?
a. Medical treatment with sublingual nitroglycerin, nitrates, or calcium-channel
blockers
b. Repeated bougie dilations
c. Injections of botulinum toxin directly into the lower esophageal sphincter
d. Dilation with a Gruntzig-type (volume-limited, pressure-control) balloon
e. Surgical esophagomyotomy

27. A 32-year-old man with a 3-year history of ulcerative colitis (UC) presents for

discussion for surgical intervention. The patient is otherwise healthy and does not
have evidence of rectal dysplasia. Which of the following is the most appropriate
elective operation for this patient?
a. Total proctocolectomy with end ileostomy
b. Total proctocolectomy with ileal pouch-anal anastomosis and diverting ileostomy
c. Total proctocolectomy with ileal pouch-anal anastomosis, anal mucosectomy, and
diverting ileostomy
d. Total abdominal colectomy with ileal-rectal anastomosis
e. Total abdominal colectomy with end ileostomy and very low Hartmann

28. A 56-year-old woman is referred to you about 3 months after a colostomy


subsequent to a sigmoid resection for cancer. She complains that her stoma is not
functioning properly. Which of the following is the most common serious
complication of an end colostomy?
a. Bleeding
b. Skin breakdown
c. Parastomal hernia
d. Colonic perforation during irrigation
e. Stomal prolapse

29. A 60-year-old male patient with hepatitis C with a previous history of variceal
bleeding is admitted to the hospital with hematemesis. His blood pressure is 80/60
mm Hg, physical examination reveals splenomegaly and ascites, and initial
hematocrit is 25%. Prior to endoscopy, which of the following is the best initial
management of the patient?
a. Administration of intravenous octreotide
b. Administration of a -blocker (eg, propranolol)

c. Measurement of prothrombin time and transfusion with cryoglobulin if elevated


d. Empiric transfusion of platelets given splenomegaly
e. Gastric and esophageal balloon tamponade (Sengstaken-Blakemore tube)
30. A 70-year-old woman has nausea, vomiting, abdominal distention,

and episodic crampy midabdominal pain. She has no history of previous


surgery but has a long history of cholelithiasis for which she has refused
surgery. Her abdominal radiograph reveals a spherical density in the right
lower quadrant. Which of the following is the definitive treatment for this
patients bowel obstruction?
a. Ileocolectomy
b. Cholecystectomy
c. Ileotomy and extraction
d. Nasogastric (NG) tube decompression
e. Intravenous antibiotics
31. A 54-year-old man presents with sudden onset of massive, painless, recurrent
hematemesis. Upper endoscopy is performed and reveals bleeding from a lesion in the
proximal stomach that is characterized as an abnormally large artery surrounded by
normal-appearing gastric mucosa. Endoscopic modalities fail to stop the bleeding.
Which of the following is the most appropriate surgical management of this patient?
a. Wedge resection of the lesion
b. Wedge resection of the lesion with truncal vagotomy
c. Wedge resection of the lesion with highly selective vagotomy
d. Wedge resection of the lesion with truncal vagotomy and antrectomy
e. Subtotal gastrectomy

31. A 53-year-old man presents to the emergency room with left lower quadrant
pain, fever, and vomiting. CT scan of the abdomen and pelvis reveals a thickened
sigmoid colon with inflamed diverticula and a 7-cm by 8-cm rim-enhancing fluid
collection in the pelvis. After percutaneous drainage and treatment with antibiotics,
the pain and fluid collection resolve. He returns as an outpatient to clinic 1 month
later. He undergoes a colonoscopy, which demonstrates only diverticula in the
sigmoid colon. Which of the following is the most appropriate next step in this
patients management?
a. Expectant management with sigmoid resection if symptoms recur
b. Cystoscopy to evaluate for a fistula
c. Sigmoid resection with end colostomy and rectal pouch (Hartmann procedure)
d. Sigmoid resection with primary anastomosis
e. Long-term suppressive antibiotic therapy
Diverticular abscesses are treated with percutaneous drainage initially followed by definitive
resectional therapy. Initial percutaneous drainage allows for a 1-stage procedure that consists of
resection of the affected colon with primary anastomosis. Perforated diverticulitis is typically treated
with either the Hartmann procedure (sigmoid resection with end colostomy and rectal stump) or
sigmoid resection, anastomosis, and diverting loop ileostomy.

32. A 52-year-old man with gastric outlet obstruction secondary to a duodenal ulcer
presents with hypochloremic, hypokalemic metabolic alkalosis. Which of the
following is the most appropriate therapy for this patient?
a. Infusion of 0.9% NaCl with supplemental KCl until clinical signs of volume
depletion are eliminated
b. Infusion of isotonic (0.15 N) HCl via a central venous catheter
c. Clamping the nasogastric tube to prevent further acid losses
d. Administration of acetazolamide to promote renal excretion of bicarbonate
e. Intubation and controlled hypoventilation on a volume-cycled ventilator to further
increase PCO2
33. A 68-year-old man presents to the ED complaining of a daily

headache for almost a month. He describes the headache as being dull,


difficult to localize, most intense in the morning, and abating in the early
afternoon. He also noticed progressive weakness of his right upper and
lower extremity. Which of the following headache syndromes are the
signs and symptoms most consistent with?
a. Headache caused by a mass lesion
!b. Cluster headache
!c. Tension-type headache!
d. Headache from intracranial hypertension
e. Waking or morning migraine

34. A 70-year-old woman presents to the ED with dark stool for 3 weeks.
She occasionally notes bright red blood mixed with the stool. Review of
systems is positive for decreased appetite, constipation, and a 10-lb
weight loss over 2 months. She denies abdominal pain, nausea, vomiting,
and fever, but feels increased weakness and fatigue. She also describes a
raspy cough with white sputum production over the previous 2 weeks.
Examination reveals she is pale, with a supine BP of 115/60 mm Hg and
HR of 90 beats per minute. Standing BP is 100/50 mm Hg, with a pulse
of 105 beats per minute. Which of the following is the most likely
diagnosis?
a. Hemorrhoids!

b. Diverticulitis!
c. Mallory-Weiss tear
d. Diverticulosis!
e. Adenocarcinoma

11.2
1. A 60-year-old woman complains of fever and constant left lower quadrant
pain of 2-day duration. She has not had vomiting or rectal bleeding. She has a
history of hypertension but is otherwise healthy. She has never had similar
abdominal pain, and has had no previous surgeries. Her only regular
medication is lisinopril. On examination blood pressure is 150/80, pulse 110,
and temperature 38.9C (102F). She has normal bowel sounds and left lower
quadrant abdominal tenderness with rebound. A complete blood count reveals
WBC = 28,000. Serum electrolytes, BUN, creatinine, and liver function tests
are normal. What is the next best step in evaluating this patients problem?
a. Colonoscopy
b. Barium enema
c. Exploratory laparotomy
d. Ultrasound of the abdomen
e. CT scan of the abdomen and pelvis
The most likely diagnosis in this patient is acute diverticulitis.

2. a child suffer a provoked bite from a stray dog that was captured
by animal control and appears healthy. the most appropriate action
would be:
a-confine and observe the dog for 10 days for sings suggestive of
rabies
b-submit the dog's heads for examination for rabies

c-begin rabies vaccination


d-administer IG rabies (HRIG) and begin rabies vaccinatione-none
of above because I s a provokes attack

3. A 48-year-old man with a past medical history of hepatitis C and cirrhosis presents
to the ED complaining of acute-onset abdominal pain and chills. His BP is 118/75
mm Hg, HR is 105 beats per minute, RR is 16 breaths per minute, temperature is
101.2F rectally, and oxygen saturation is 97% on room air. His abdomen is
distended, and diffusely tender. You decide to perform a paracentesis and retrieve 1 L
of cloudy fluid. Laboratory analysis of the fluid shows a neutrophil count of 550
3
cells/mm . Which of the following is the most appropriate choice of
treatment?!
a. Metronidazole!
b. Vancomycin!
c. Sulfamethoxazole/trimethoprim (SMX/TMP)
d. Neomycin and lactulose!
e. Cefotaxime

4. A 22-year-old man presents to the ED complaining of dysuria for 3 days. He states


that he has never had this feeling before. He is currently sexually active and uses a
condom most of the time. He denies hematuria but notes a yellowish discharge from
his urethra. His BP is 120/75 mm Hg, HR is 60 beats per minute, and temperature is
98.9F. You send a clean catch urinalysis to the laboratory that returns positive for
leukocyte esterase and 15 white blood cells per high power field (WBCs/hpf). Which
of the following is the most appropriate next step in management?

a. Send a urethral swab for culture and administer 125 mg ceftriaxone intramuscularly
and 1 g azithromycin orally.
b. Send urine for culture and administer SMX/TMP orally.!
c. Discharge the patient with strict instructions to return if his symptoms worsen.
d. Order a CT scan to evaluate for a kidney stone.
e. Have him follow-up immediately with a urologist to evaluate for testicular cancer.

5. An 81-year-old woman presents to the ED after tripping over the sidewalk curb
and landing on her chin causing a hyperextension of her neck. She was placed in a
cervical collar by paramedics. On examination, she has no sensorimotor function of
her upper extremities. She cannot wiggle her toes, has 1/5 motor function of her
quadriceps, and only patchy lower extremity sensation. Rectal examination reveals
decreased rectal tone. Which of the following is the most likely diagnosis?
a. Central cord syndrome!
b. Anterior cord syndrome!
c. Brown-Squard syndrome!
d. Transverse myelitis!
e. Exacerbation of Parkinson disease

6. A 22-year-old man presents to the ED after being ejected from his vehicle
following a high-speed motor vehicle collision. Upon arrival, his BP is 85/55 mm Hg
and HR is 141 beats per minute. Two large-bore IVs are placed in the antecubital
veins and lactated Ringer solution is being administered. After 3 L of crystalloid fluid,
the patients BP is 83/57 mm Hg. Which of the following statements is most
appropriate regarding management of a hypotensive trauma patient who fails to
respond to initial volume resuscitation?
a. It is important to wait for fully cross-matched blood prior to transfusion.
b. Whole blood should be used rather than packed red blood cells (RBCs).
c. Blood transfusion should begin after 4 L of crystalloid infusion.!
d. Type O blood that is Rh-negative should be transfused.

e. Type O blood that is Rh-positive should be transfused.

7. A 17-year-old adolescent boy is found unconscious in a swimming pool. He is


brought into the ED by paramedics already intubated. In the ED, the patient is
unresponsive with spontaneous abdominal breathing at a rate of 16 breaths per
minute, BP 80/50 mm Hg, and HR 49 beats per minute. In addition to hypoxemia,
what condition must be considered earliest in the management of this patient?
a. Cervical spine injury
a. Cervical spine injury
b. Electrolyte imbalance
c. Metabolic acidosis
d. Severe atelectasis
e. Toxic ingestion
8. A 61-year-old man presents to the ED with low back pain after slipping on an icy

sidewalk yesterday. He states that the pain started on the left side of his lower back
and now involves the right and radiates down both legs. He also noticed difficulty
urinating since last night. On neurologic examination, he cannot plantar flex his feet.
Rectal examination reveals diminished rectal tone. He has a medical history of
chronic hypertension and underwent a vessel surgery many years earlier. Which of
the following is the best diagnosis?
a. Abdominal aortic aneurysm (AAA)
b. Disk herniation!
c. Spinal stenosis!
d. Cauda equina syndrome
e. Osteomyelitis
leg pain, saddle anesthesia, and impaired bowel and bladder function (retention or
incontinence).
9. You have just prescribed phenytoin for a 12- year-old boy with new onset

of epilepsy. Of the following side effects, which is most likely to occur in this
patient?
(A) lymphoma syndrome !
(B) Raynaudphenomenon !
(C) acute hepatic failure !

(D) gingival hyperplasia !


(E) opticatrophy !

10. A previously well 12-year-old girl presents to clinic because of painful swellings on the
front of the legs of about 3 days duration. Examination reveals tender erythematous nodules,
12 cm in diameter, on the extensor surfaces of the lower legs. The remainder of the physical
examination is unremarkable. Which of the following is most likely to confirm the cause of
this condition?
(A) stool smear and culture !
(B) urine analysis and BUN !
(C) throat culture !
(D) slit-lamp examination of the eye !
(E) echocardiogram !
To answer this question you must not only identify the rash as erythema nodosum (an
uncommon but not rare disease) but you must also know that group A -hemolytic
streptococcal infection is a common cause. Erythema nodosum is a reactive phenomenon
characterized by tender, erythematous nodules 12 cm in diameter. The lesions usually are on
the extensor surfaces of the extremities and are more common on the legs.

11.A 6-year-old girl presents with unilateral nonpainful, nonsuppurative conjunctivitis and
preauricular lymphadenitis. What is the most likely
causative organism?
(A) Mycobacterium avium !
(B) Bartonella henselae !
(C) Adenovirus !
(D) Staphylococcus aureus !
(E) Chlamydia trachomatis !
12. Causes of macrocephaly include all of the following EXCEPT:
A. Klinefelter syndrome
B. Chronic subdural effusions
C. Hydrocephalus
D. Familial factors
E. Congenital CMV infection

13. A 2-month-old infant has severe dyspnea and cyanosis.

Chest roentgenogram reveals minimal cardiomegaly and a


diffuse reticular pattern of the lung fields. Which of the
following best explains these findings?
(A) acute viral myocarditis !
(B) hypoplastic left heart syndrome !
(C) pulmonary artery atresia
(D) total anomalous pulmonary drainage with venous
obstruction !
(E) transposition of the great arteries !

14. You suspect the diagnosis of a brain tumor in a 2-year-old girl with a recent
history of ataxia, slurred speech, and early morning vomiting. Which statement
about childhood brain tumors is true?
(A) Most are located in the midline and/or below the tentorium cerebri. !
(B) Brain tumors are a rare type of cancer in childhood. !
(C) Signs of increased intracranial pressure are rare on presentation. !
(D) Seizures are the presenting complaint in most cases. !
(E) Most cases occur in the first year of life. !

16. All of the following antiviral medications are correctly matched


with a significant side effect EXCEPT:
A. Acyclovirthrombotic thrombocytopenic purpura
B. Amantadineanxiety and insomnia
C. Foscarnetacute renal failure
D. Ganciclovirbone marrow suppression
E. Interferonfevers and myalgias

17. A 3-year-old boy presents with acute right leg pain and a limp. There is
no history of trauma. He holds his right hip in external rotation and flexion
and he has mild restriction of range of motion. He appears otherwise well
and is afebrile. His WBC is normal and ESR is 25 mm/h. What is the best
treatment option at this time?
(A) intravenous antibiotics !
(B) surgical drainage of the right hip joint !
(C) anti-inflammatory drugs and bed rest !
(D) oral antibiotics !
(E) intra-articular corticosteroids !
(C) Transient synovitis is the most common cause for limping in a healthy
child at this age. Transient synovitis classically occurs 12 weeks after a
nonspecific upper respiratory tract infec- tion. Transient synovitis is a
diagnosis of exclusion. Treatment of this disorder is symptomatic with
nonsteroidal anti-inflammatory medications.
18. You counsel the new parents of a baby boy with hypospadias

about circumcision. Which information will you most likely share


with the parents?
(A) There is clearly an increased risk for penile cancer in
uncircumcised males.
(B) Urinary tract infections are 1015 times more common in
uncircumcised infants.
(C) Circumcision reduces the risk of sexually transmitted diseases.
(D) Complications following circumcision are very rare.
(E) Circumcision can be safely done in infants with hypospadias.

19. All of the following clinical findings are consistent with the
diagnosis of molluscum contagiosum EXCEPT:
A. Involvement of the genitals
B. Involvement of the soles of the feet
C. Lack of inflammation or necrosis at the site of the
rash
D. Rash associated with an eczematous eruption
E. Rash spontaneously resolving over 3 to 4 months

20. An 8-year-old boy is referred for new-onset seizures. Which of the following
would mostly support a diagnosis of complex partial (psychomotor) seizures?
(A) normal mental state, consciousness, and responsiveness during seizure
(B) a brief tonic-clonic phase
(C) automatisms
(D) three-persecond spike-and-wave pattern on EEG
(E) normal mental state, consciousness, and responsiveness after seizure

21. The parents of a 3-yr-old girl with a history of several previous coughing

and wheezing exacerbations are wondering if their toddler is likely to develop


persistent asthma. Which of the following is a strong riskfactor for persistent
asthma in toddlers with recurrent wheezing
A. Eczema
B. Colic
C. Living on a farm
D. Female gender
E. Otitis media with effusion

22. Which of the following is most commonly associated with tricyclic


antidepressant toxicity?
a) 3rd degree AV block
b) PR interval prolongation
c) QT interval prolongation
d) Widened QTe) T elevation
23. A 68-year-old woman with a prior diagnosis of asthma presents

to your clinic for a routine clinic visit. She complains of occasional


palpitations and tremor. Her dyspnea is well controlled. Her past
medical history is remarkable for hospitalization for mild congestive
heart failure 2 months ago; she notes occasional postprandial acid
reflux. Her medications include lisinopril, digoxin, furosemide, an
intermittent short-acting inhaled beta agonist, and theophylline.
She uses an over-the-counter pill (whosename she cannot
remember) for the reflux symptoms. On examination her heart rate
is 112 beats/minute.S1 and S2 are normal; she has a mild tremor
of the outstretched hands. What is the best next step in
hermanagement?
a. Chest x-ray to rule out exacerbation of congestive heart failure
b. Theophylline level
c. Spirometry before and after bronchodilator
d. Intermittent lorazepam 0.5 mg po tid
e. Discontinue beta agonist and substitute inhaled ipratropium

You are evaluating a small-for-gestational-age infant for


tachycardia. There is no prenatal care history available, but based
on the infants signs and symptoms and the obstetricians
description of the mother you suspect neonatal thyrotoxicosis.
Thyrotoxicosis in the first day of life most likely occurs in an infant
born to a mother with which of the following diagnoses?
(A) with untreated hypothyroidism
(B) with untreated Graves disease
(C) with Graves disease being treated with antithyroid medications
24.

(D) with euthyroid goiter


(E) receiving iodides as therapy for chronic bronchitis

25. An infant delivered precipitously to a mother who had not


received prenatal care. The infant has finding suggestive of
congenital syphilis. Maternal and infant testing confirm this
diagnosis. Which of the following is most suggestive of early
congenital syphilis?
(A) disseminated intravascular coagulation
(B) bullous lesions of the palms and soles
(C) hepatitis
(D) dermal erythropoiesis
(E) pneumonia

26. A newborn infant has micrognathia, glossoptosis, and cleft soft


palate. These findings are consistent with Pierre Robin sequence.
Which of the following life-threatening events is associated with
these findings?
(A) heart failure
(B) seizures
(C) intestinal obstruction
(D) metabolic acidosis
(E) upper airway obstruction

27. A 64-year-old man complains of cough, increasing shortness of breath, and


headache for the past 3 weeks. He has mild hypertension for which he takes
hydrochlorothiazide; he has smoked 1 pack of cigarettes a day for 40 years. On
examination you notice facial plethora and jugular venous distension to the angle of
the jaw. He has prominent veins over the anterior chest and a firm to hard right
supraclavicular lymph node. Cardiac examination is normal and lungs are without
rales. Peripheral edema is absent. What is the most likely cause of his condition?
a. Long-standing hypertension

b. Gastric carcinoma
c. Emphysema
d. Lung cancer
e. Nephrotic syndrome

A 5-day-old infant boy is jaundiced. The total bilirubin level is 14


mg/dL and the direct bilirubin is 4 mg/dL. Which of the following
tests is the most appropriate?
(A) blood type and direct antibody test on the infants blood
(B) blood type and direct antibody test on the mothers blood
(C) urine analysis and culture
(D) hepatitis serology
28.

(E) examination of infants blood smear

29. A 52-year-old man with cirrhosis resulting from chronic hepatitis C


presents with increasing right upper quadrant pain, anorexia, and 15-lb weight
loss. The patient is mildly icteric and has moderate ascites. A friction rub is
heard over the liver. Abdominal paracentesis reveals blood-tinged fluid, and
CT scan shows a 4-cm solid mass in the right lobe of the liver. Which of the
following is the most important initial diagnostic study?
a. Serum -fetoprotein level
b. Colonoscopy to search for a primary neoplasm
c. Measurement of hepatitis C viral RNA
d. Upper GI endoscopy
e. Positron emission tomography scan

30. A 2-month-old infant has severe dyspnea and

cyanosis. Chest roentgenogram reveals minimal


cardiomegaly and a diffuse reticular pattern of
the lung fields. Which of the following best
explains these findings?
(A) acute viral myocarditis
(B) hypoplastic left heart syndrome

(C) pulmonary artery atresia


(D) total anomalous pulmonary drainage with venous obstruction
(E) transposition of the great arteries
31. A 38-year-old man with HIV/AIDS presents with 4 weeks of
diarrhea, fever, and weight loss. Which of the following tests
makes the diagnosis of cytomegalovirus (CMV) colitis?
A. CMV IgG
B. Colonoscopy with biopsy
C. Serum CMV polymerase chain reaction
D. Stool CMV antigen
E. Stool CMV culture
32. A 23-year-old previously healthy female letter car- rier works in
a suburb in which the presence of rabid foxes and skunks has
been documented. She is bitten by a bat, which then flies away.
Initial examination reveals a clean break in the skin in the right
upper forearm. She has no history of receiving treatment for rabies
and is unsure about vaccination against tetanus. The physician
should:
A. Clean the wound with a 20% soap solution.
B. Clean the wound with a 20% soap solution andadminister
tetanus toxoid.
C. Clean the wound with a 20% soap solution, admin-ister tetanus
toxoid, and administer human rabiesimmune globulin
intramuscularly.
D. Clean the wound with a 20% soap solution, adminis-ter tetanus
toxoid, administer human rabies immune globulin IM, and
administer human diploid cell vaccine.
E. Clean the wound with a 20% soap solution and admin- ister
human diploid cell vaccine.

33. A 64-year-old woman who is receiving chemotherapy for metastatic breast cancer has
been treating midthoracic pain with acetaminophen. Over the past few days she has become
weak and unsteady on her feet. On the day of admission she develops urinary incontinence.
Physical examination reveals fist percussion tenderness over T8 and moderate symmetric
muscle weakness in the legs. Anal sphincter tone is reduced. Which of the following
diagnostic studies is most important to order?
a. Serum calcium!

b. Bone scan!
c. Plain radiographs of the thoracic spine
d. MRI scan of the spine!
e. Electromyogram with nerve conduction studies
Spinal cord compression is an oncologic emergency. Major neurological deficit is often
irreversible and severely compromises the patients remaining quality of life. Vertebral and
then epidural involvement precede the neurological findings; the thoracic cord is involved
70% of the time. The patient is often given high-dose dexamethasone before being sent for
MRI. In the presence of neurological compromise, the definitive test, MRI scan, should be
performed as quickly as possible.

34. A 15-month-old boy has strabismus and a white pupillary reflex.

Ophthalmologic exam- ination reveals a white retinal mass. This patient is at


increased risk for development of which other tumor?
(A) leukemia !
(B) lymphoma !
(C) osteosarcoma !
(D) Ewing sarcoma !
(E) rhabdomyosarcoma !

35. An 18-year-old boy presents with acute severe chest pain. EKG and enzyme studies

confirm an acute myocardial infarction. Cardiac catherization reveals a coronary


artery aneurysm !with thrombosis. Which constellation of symptoms in his past could
explain this finding?

(A) conjunctivitis, fever, cervical lymphadenopathy !


(B) meningitis, conjunctivitis, pallor !
(C) cervical lymphadenopathy, hepatitis, rash !

(D) fever, irritability, pancreatitis !


(E) hepatosplenomegaly, rash, conjunctivitis

36. Which of the following is true regarding community acceured pneumonia?


A. Blood culture should be drown prior to initiation of antibiotic treatment.
B. empiric treatment should be initiated without need to identify the organism
C. Sputum culture should be drown prior to initiation of antibiotic treatment
D. chest CT should be performed to diagnosis pneumonia

37. A 3-year-old girl develops petechiae and bruises on her

extremities while she is recov- ering from a cold. She is brought to


medical attention after she has a transient nosebleed. Physical
examination shows a toddler with widespread petechiae and
bruising who oth- erwise looks healthy. What is the best treatment for this patient?
(A) plasmapheresis
(B) intravenous gammaglobulin
(C) vincristine and methotrexate
(D) intravenous antibiotics
(E) platelet transfusion

38. A 33-year-old carpenter was working on a construction project to build a new


house. When using a high-pressure paint gun, he inadvertently injected his left index
finger. On arrival to the ED, he complains of intense hand pain. On examination, you
note a 2-mm wound over the second proximal phalange. He has full range of motion
and brisk capillary refill. Radiographs of the finger show soft tissue swelling, a small
amount of subcutaneous air, but no fracture. His tetanus is up to date. Which of the
following is the most appropriate disposition for this patient?
a. Place the hand in a radial gutter splint and have the patient follow up with an
orthopedic surgeon in 1 week.
b. Discharge home with pain medication and have the patient return for repeat

radiographs in 1 week.
c. Order a CT scan of the finger to confirm that there is no occult fracture before
discharging the patient home.
d. Place the hand in a radial gutter splint, prescribe a 10-day course of antibiotics,
and have the patient follow up with an orthopedic surgeon in 1 week.
e. Place the hand in a radial gutter splint, administer broad-spectrum antibiotics, and
admit the orthopedic service for operative debridement.

A 72-year-old female with severe osteoporosis presents for


evaluation of shortness of breath. She is a lifetime nonsmoker and
has had no exposures. On physical examination you note marked
kyphoscoliosis. All the following pulmonary abnormalities are
expected except?
A. Restrictive lung disease
B. Alveolar hypoventilation
C. Obstructive lung disease
D. Ventilation-perfusion abnormalities with hypoxemiaE.
Pulmonary hypertension
39.

40. A 75 year-old man is complaining that for several months he


has been experiencing increasing fatigue. He has a history of
hypertension, gastroesophageal reflux disease, and large
diaphragmatic hernia. He has not experienced any weight loss or
other general symptoms.On physical examination, conjunctival
pallor is noted. A fecal occult blood test(FOBT) is positive.
Laboratory testing reveals iron deficiency anemia. Which of the
following is most likely cause of this patients anemia?
a. Gastric cancer.
b. Colorectal cancer.
c. Angiodysplasia.
d. Peptic ulcer disease.
e. Diaphragmatic hernia and esophagitis.

41.A 8 year old child referred to the clinic because of bed wetting,
several behavioral interventions have been attempted, including
eliminated fluid intake in the evening, scheduled awakenings at
nights to use the bathroom and urine alarm (bell and pad). These
techniques were unsuccessful and the child continued to urinate
every night. Which medication is the most appropriate to prescribe
to treat the enuresis:
A. retalin

B. paroxetin
C. trazodone
D. imepramin (primonil)
E. benztropine (cogentin)

42. a full term neonate is noted to have circumoral cynosis and


twitching of his left hand at 12 hours of age on physical
examination he is found to have an absent pupillary response to
light and a small penis which of the following is the most likely
diagnosis:
a .hypocalcemia
b . hypoglycemia
c . congenital hypothyroidism
d . congenital heart disease
e . idiopathic epilepsy

43. A teenage boy falls from his bicycle and is run over by a truck.On arrival in the emergency
room(ER),he is awake and alert and appears frightened but in no distress. The chest
radiograph suggests an air-fluid level in the left lower lung field and the
nasogastric tube seems to coil upward into the next best step in his management?
a)Placement of a left chest tube

b)Thoracotomy
c)Laparotomy
d)Esofagogastroscopy
e)Diagnostic peritoneal lavage

12.2

1. In the ICU, a patient suddenly becomes pulseless and

unresponsive, with cardiac monitor indicating


ventricular tachycardia. The crash cart is immediately available.
What is the best first therapy?

a. Amiodarone 150-mg IV push


b. Lidocaine 1.5-mg/kg IV push
c. Epinephrine 1-mg IV push
d. Defibrillation at 200 J
e. Defibrillation at 360 J

2. A newborn boy was diagnosed prenatally with bilateral


hydronephrosis, distended bladder, and oligohydramnios. What
will be the most likely diagnosis?
(A) urethral strictures
(B) anterior urethral valves(
C) prune-belly syndrome
(D) posterior urethral valves
(E) meatal stenosis

3. Of the following steps taken prior to the induction of general anesthesia, which
has been the key factor in decreasing maternal mortality rates from general
anesthesia?
a. Antacids!
b. Preoxygenation!
c. Uterine displacement
!d. Aggressive IV hydration
When you perform Anesthesia there is a risk of pulmonary aspiration. So you
give antacids to increase the stomach PH.

4. Which of the following is the LEAST important to the normal progress of labor?
a. Cervical position
b. Uterine contractions
!c. Resistance of maternal tissues

!d. Maternal intra-abdominal pressure

5. What is the most common site of tubal pregnancy implantation?

a. Cornua
b. Fimbria
c. Isthmus
d. Ampulla
6. Which of the following is NOT a sonographic fetal finding of congenital syphilis?
a. Hepatomegaly
b. Lymphadenopathy
c. Placentomegaly
d. Ascites/hydrops

7. In pregnancies complicated by an infectious cause of preterm

labor, which of the following routes of initial infection is considered


most common ?
a. Ascension of vaginal bacteria
b. Transplacental transfer of maternal infection
c. Retrograde transmission through fallopian tubes
d. Iatrogenic inoculation during transamnionic procedures

8. Drugs whose efficacy may be diminished by combination oral contraceptive pill

use include which of the following?


a. Lisinopril
!b. Macrodantin!
c. Acetaminophen
d. Metoclopramide

10. A number of antibiotic regimens have been used to prolong the

latency period in women with preterm ruptured membranes who


are attempting expectant management. Which antibiotic should be
avoided in this setting since it has been associated with an
increased risk of necrotizing enterocolitis in the newborn?
a. Amoxicillin
b. Amoxicillin-clavulanate
c. Ampicillin
d. Erythromycin
11. Witch of the following is the best choice to prepare a patient
with type 1 VON WILLERBRAND disease for surgery?
A. recombinant (pure ) factor XIII
B. VON WILLERBRAND factor
C.factor XIII
D. desmopressin

12. Recurrent midtrimester pregnancy loss in the setting of


painless cervical dilatation is termed:
a. Cervical incompetence
b. Cervical ineptitude
c. Cervical deficiency
d. Cervical ineffectiveness

13. the most common primary site of carcinoid tumor is :


A- the appendix
B - Bronchus
C- rectum
D- Small intestines

14. A teenage boy falls from his bicycle and is run over by a truck.On arrival in

the emergency room(ER),he is awake and alert and appears frightened but in
no distress. The chest radiograph suggests an air-fluid level in the left lower
lung field and the nasogastric tube seems to coil upward into the next best
step in his management?
a)Placement of a left chest tube
b)Thoracotomy
c)Laparotomy
d)Esofagogastroscopy
e)Diagnostic peritoneal lavage
The patient has an acute diaphragmatic rupture, which occurs in about 4% of
patients who sustain either blunt abdominal or chest trauma, and should be
treated with immediate laparotomy, which allows both for examination of the
intra-abdominal solid and hollow viscera for associated injuries and for
adequate exposure of the diaphragm to allow secure repair. Because of the
risk of vascular compromise of the contents of the hernia, exacerbated by the
negative thoracic pressure, acute diaphragmatic rupture should be repaired
immediately. Diagnosis may be difficult. The finding of an air-fluid level in the
left lower chest, with a nasogastric tube entering it after blunt trauma to the
abdomen, is diagnostic of diaphragmatic rupture with gastric herniation into
the chest. Esophagogastroscopy is of limited value. CT scanning and MRI
may be useful adjuncts, but neither can definitively rule out diaphragmatic
rupture. Diagnostic peritoneal lavage is neither sensitive nor specific for
diaphragmatic injuries, particularly in the absence of significant hemorrhage.
Diaphragmatic repair can be accomplished via the left chest, but laparotomy
is the procedure of choice for acute traumatic rupture for the stated reasons.

15. Although an effective method of contraception, pregnancy in a patient following


tubal sterilization is often complicated by which of the following?
a. Twinning!
b. Placenta previa!

c. Placenta accreta!
d. Ectopic implantation

16. A 47-year-old woman complains of fatigue, weight loss, and

itching after taking a hot shower. Physical examination shows


plethoric facies and an enlarged spleen, which descends 6 cm
below the left costal margin. Her white cell count is 17,000 with a
normal differential, the platelet count is 560,000, and hemoglobin
is 18.7. Liver enzymes and electrolytes are normal; the serum uric
acid level is mildly elevated. What is the most likely underlying
process?
a. Myelodysplastic syndrome
b. Myeloproliferative syndrome
c. Paraneoplastic syndrome
d. Cushing syndrome
e. Gaisbck syndrome

17. A 73-year-old woman is admitted for deep venous thrombosis


and concern for pulmonary embolism. She has a history of type 2
diabetes mellitus, hypertension, and coronary artery disease. She
had been admitted for a three-vessel coronary artery bypass graft

2 weeks prior to this admission. She did well and was dismissed 5
days after the procedure. Pain and swelling of the right leg began
2 days before this admission; she has noticed mild dyspnea but no
chest pain. The clinical suspicion of deep vein thrombosis (DVT) is
confirmed by a venous Doppler, and the patient is started on
unfractionated heparin. Her initial laboratory studies, including
CBC, are normal.The next day her pain has improved, and helical
CT scan of the chest reveals no evidence of pulmonary embolism.
She is instructed in the use of low-molecular-weight heparin and
warfarin; she is eager to go home. Her serum creatinine is normal.
Her predischarge CBC shows no anemia, but the platelet count
has dropped to 74,000. An assay for antibodies to heparin-platelet
factor 4 complexes is ordered. What is the best next step in her
management?
a. Dismiss the patient on low-molecular heparin, warfarin, and
close outpatient follow-up.
b. Obtain a liver-spleen scan to look for platelet sequestration.
c. Discontinue all forms heparin, continue warfarin, and add aspirin
162 mg daily until INR becomes therapeutic.
d. Keep the patient in the hospital, discontinue unfractionated
heparin, add low-molecular-weight heparin, and monitor the
platelet count daily.
. Keep the patient in the hospital, discontinue all forms of
e
heparin, and start the patient on lepirudin by intravenous infusion.

17. A 70-year-old man complains of 2 months of low back pain and


fatigue. He has developed fever with purulent sputum production.
On physical examination, he has pain over several vertebrae and
rales at the left base. Laboratory results are as follows:

Hemoglobin: 7 g/dL
MCV: 89 fL (normal 86-98) WBC: 12,000/mL
BUN: 44 mg/dLCreatinine: 3.2 mg/dL
Ca: 11.5 mg/dLChest x-ray: LLL infiltrate Reticulocyte count: 1%
The definitive diagnosis is best made by which of the following?
a. 24-hour urine protein
b. Bone scan
c. Renal biopsy
d. Rouleaux formation on blood smear
e. Greater than 30% plasma cells in the bone marrow
Multiple myeloma would best explain this patients presentation. The onset of myeloma is
often insidious. Pain caused by bone involvement, anemia, renal insufficiency, and bacterial
pneumonia often follow. This patient presented with fatigue and bone pain, then developed
bacterial pneumonia probably secondary to Streptococcus pneumoniae, an encapsulated
organism for which antibody to the polysaccharide capsule is not adequately produced by the
myeloma patient. There is also evidence for renal insufficiency. Hypercalcemia is frequently
seen in patients with multiple myeloma and may be life threatening. Definitive diagnosis of
multiple myeloma is made by demonstrating greater than 30% plasma cells in the bone
marrow

18. The operation most appropriate for severe stress ulcer bleeding is

a.Vagotomy and pyloroplasty !


b.Highly selective vagotomy !
c.Near-total or total gastrectomy !
d.Ligation of the bleeding sites !
e.Antrectomy with Billroth II reconstruction !

19. the most appropriate operation of a 30-year old patient with a history of
peptic ulcer disease who presents with a perforated duodenal ulcer is
a) Patch closure with a highly selective vagotomy
b) Gastrectomy with a Billroth I reconstruction

c) Gastrectomy with a Billroth II reconstruction


d) Vagotomy and pyloroplastye) Gastrojejunostomy and Omeprazol

20. Which of the following is true regarding adenomatous colonic

polyps.:
a. Polyps less than 1 cm in diameter are unlikely to be malignant
b. Villous adenomas most commonly occur in the cecum
c. FAP (familial adenomatous polyposis; is an autosomal recessive
condition
d. Hamartomatous polyps have a high incidence of malignant
transformatione. Fecal occult blood testing has not changed the
mortality from colon cancer.

21. The obstetrician performing the intrauterine transfusion is


counseling the mother regarding complications of this procedure.
Which of the following is the most common complication of
intrauterine transfusion?
(A) a transfusion reaction (mismatch)
(B) graft-versus-host reaction
(C) premature onset of labor
(D) acquired immunodeficiency syndrome (AIDS)
(E) renal failure

22. A 66-year-old postmenopausal woman presents with a


painless breast mass and is found to have a 3-cm infiltrating ductal
breast cancer. Sentinel node sampling reveals metastatic cancer
in the sentinel node; a formal axillary node dissection shows that 4
of 13 nodes are involved by the malignant process. Both estrogen
and progesterone receptor are expressed in the tumor. There is no
evidence of metastatic disease outside the axilla. In addition to
lumpectomy and radiation therapy to the breast and axilla, what
should her treatment include next?
a. No further treatment at this time
b. Radiation therapy to the internal mammary nodes
c. Platinum-based adjuvant chemotherapy
d. Bilateral oophorectomy
e. Adjuvant hormonal therapy (tamoxifen or aromatase inhibitor)

23. During a routine prenatal visit a mother states she has been
reading about kernicterus. She is very concerned that her infant
will develop kernicterus if she provides breast milk for the infant
since she has also read that some breastfed infants have high
bilirubin levels. Which of the following is most predictive for the
development of kernicterus?
(A) hyperbilirubinemia within the first 24 hours of life
(B) peak conjugated bilirubin level
(C) peak unconjugated bilirubin level
(D) duration of hyperbilirubinemia
(E) hemoglobin level immediately after birth
24. Prevention of bilirubin encephalopathy or kernicterus is one of
the goals for the appropriate diagnosis and treatment of
hyperbilirubinemia. Which of the following mechanisms has a role
in preventing these adverse outcomes?
(A) Unconjugated bilirubin is not lipid soluble.
(B) Unconjugated bilirubin is tightly bound to albumin.

(C) Unconjugated bilirubin is tightly bound to hemoglobin.


(D) The bloodbrain barrier is impermeable to unconjugated
bilirubin.
(E) Unconjugated bilirubin is rapidly metabolized by cerebrospinal
fluid.

25. Which is the most likely cause of intussusception in a 20-yearold man?


a. Idiopathic.
b. Lymphoma of the small bowel.
c. Carcinoid.
d. Adhesions.
26. A newborn infant with stigmata of Down syndrome has a heart
murmur. Which of the following cardiac lesions is most likely in this
baby?
(A) hypoplastic left heart syndrome
(B) total anamolous venous return
(C) coarctation of the aorta
(D) anamolous coronary artery
(E) atrioventricular defect

27. Patients who have suffered a penetrating colon injury involving


more than 50% of the rectal wall should undergo which of the
Following:
a. A diverting colostomy
b. Exteriorization of the injured segment
c. Primary repair of the injury
d. Segmental resection and anastomosis
e. Hartmann's procedure and drainage
28. In which of the following cases might internal podalic version be

indicated?
(A) vertex delivery of the first twin and breech presentation of the
second twin
(B) term transverse lie with cervix completely dilated and
membranes intact

(C) double footling breech


(D) impacted shoulder presentation
(E) compound presentation

29. A 30-year-old methamphetamine user presents to L&D in


active labor. She has had no prenatal care, but says she is 912
months. You check fetal position and feel face and nose. You are
concerned, because the most common associated condition with a
face presentation is which of the following?
(A) anencephaly
(B) hydrocephaly
(C) prematurity
(D) placenta previa
(E) oligohydramnios
30. A 28-year-old G3P2002 patient presents at 38 weeks
gestation. A fetus was felt to be in breech position as judged by
information gained through Leopolds maneuvers. The fetus was
well down in the pelvis, and the uterus was irritable. Pelvimetry
was within normal limits, and the estimated fetal weight was 712 lb
and ultrasound confirms a frank breech with a well-flexed head
and AFI of 14 cm. Assuming the provider has adequate experience
in each of the following, what would not be offered to the patient as
an approach to delivery management?
(A) cesarean section
(B) external cephalic version
(C) internal podalic version
(D) vaginal breech delivery
(E) expectant management
31. Transverse lie in a multipara at term in labor is best treated by
which of the following?
(A) external version
(B) internal version and extraction
(C) oxytocin induction
(D) cesarean delivery
(E) abdominal support to effect position change

32. After a low forceps delivery for fetal intolerance, a mother


continues to have excessive vaginal bleeding despite a wellcontracted uterus. Which of the following locations is most likely to
be the site of a vaginal laceration after an instrumented delivery?
(A) extending off the cervix
(B) anterior upper third under the pubic symphysis
(C) posterior upper third from an incompletely evacuated rectum
(D) lateral middle third over the ischial spines
(E) posterior middle third over the coccyx

13.2

1. A 57-year-old woman with a history of a surgically corrected


atrial septal defect in childhood presents to the emergency
department with palpitations for 3 days. She is found to have a
heart rate of 153 beats/min and blood pressure of 128/75 mmHg,
and an ECG shows atrial flut- ter. An echocardiogram
demonstrates moderate right and left atrial dilation, postoperative
changes from her surgery, and normal left and right ventricular

function. Which of the following is true?


A. Anticoagulation with dabigatran should be initiated.
B. If a transesophageal echocardiogram does not demonstrate left
atrial thrombus, she may be cardiovertedwithout anticoagulation.
C. Intravenous heparin should be started immediately.
D. She should be immediately cardioverted.
E. Transthoracic echocardiogram is adequate to ruleout the
presence of left atrial thrombus
The patient is hemodynamically stable and has no indications for acute cardioversion.
Dabigatran is not currently FDA approved for atrial flutter. Intravenous heparin
should be started immediately if there are no contraindications, given the greater than
12-hour duration of symptoms.

2. A 37-year-old man who is healthy aside from a prior knee

surgery is evaluated in the emergency department for palpitations


that developed suddenly while eating dinner. He is found to have a
heart rate of 193 beats/min, blood pressure of 92/52 mmHg, and
normal oxygen saturation. His physical examination is normal
aside from tachycardia and mild diaphoresis. An ECG obtained
before his knee surgery shows delta waves in the early precordial
leads. His current ECG shows wide complex tachycardia. Which of
the following therapies is contraindicated for treatment of his
tachyarrhythmia?
A.Adenosine
B.Carotid sinus massage
C.DC cardioversion
D.Digoxin
E. Metoprolol

- Methylene blue or indigo carmine instilled into the bladder in a retrograde


fashion can be visualized leaking through the fistula into the vagina. If the
fistula is very small and difficult to visualize, a tampon can be placed in the
vagina following the instillation of dye; the blue dye will stain the tampon if a

vesicovaginal fistula is present. To diagnose a ureterovaginal fistula, indigo


carmine is given intravenously. As the com- pound is filtered through the
kidneys and passes through the ureters, it will stain the tampon. If a
ureterovaginal fistula is present, the retrograde dye test will be negative and
the IV dye test will be positive.

3. Even after menopause, most women have circulating estrogen. In high enough levels, this
can promote the development of endometrial cancer. It mainly originates from the
aromatization of which of the following?
(A) androstenedione to estradiol by ovarian granulosa cells
(B) androstenedione to estrone by ovarian thecal cells
(C) androstenedione to estrone by adipose tissue
(D) estradiol to estrone by adipose tissue
(E) testosterone to estradiol by adipose tissue

4. Women perceive the menstrual flow as an indication that the reproductive system
is functioning well. In fact the actual menstrual flow is associated with which of the
following?
(A) prolonged maintenance of estrogen
(B) prolonged maintenance of progesterone
(C) withdrawal of FSH
(D) withdrawal of LH
(E) withdrawal of progesterone

5. Which of the following hormone(s) is/are produced by the corpus luteum?


(A) progesterone only
(B) progesterone and estrogen only
(C) progesterone, estrogen, and inhibin only
(D) progesterone, estrogen, inhibin, and relaxin only
(E) progesterone, estrogen, inhibin, relaxin, and contractin

6. Which of the following statements best describes the role of FSH in

menstruation?
(A) FSH increases its own receptor numbers on theca cells.
(B) FSH induces granulosa cell LH receptors within the dominant follicle.
(C) FSH induces theca cell aromatase.
(D) FSH stimulates follicular growth only in the early preantral stage.
(E) FSH stimulates granulosa cell androgen production.

7. A 24-year-old patient presents with amenorrhea for more than 6 months. Prior to
the amenorrhea she had had oligomenorrhea for a number of months. She is not
pregnant, her weight has been stable with body mass index (BMI) of 23, and there is
no evidence of hirsutism or galactorrhea. On the basis of clinical evaluation, it
appears that she has GnRH suppression. A detailed history needs to be considered
regarding medication use since the inhibitory action of sex steroids on GnRH
secretion is primarily mediated by which of the following?
(A) dopamine
(B) melatonin
(C) norepinephrine
(D) opioid peptides
(E) serotonin

8. A 7-year-old girl is brought in for evaluation. On examination, she has welldeveloped pubic hair and breasts and she is 99% of height for her age. Her mother
recently noted some bloodstains on her underwear. Which of the following conditions
is most likely the cause of these findings?
(A) estrogen-producing ovarian cyst
(B) hepatoma

(C) hypothalamic tumor


(D) sex steroid-containing medication
(E) thecal/Leydig cell tumor
9. A 16-year-old girl is brought to clinic by her mother with concerns regarding the lack of
any signs of puberty. The appearance of external genitalia is of a normal prepubertal female.
Laboratory studies show markedly elevated FSH and LH levels. Which of the following
causes of delayed puberty accompanies elevated circulating gonadotropin levels?
(A) chronic illness
(B) gonadal dysgenesis
(C) hypothalamic tumors
(D) Kallmann syndrome
(E) malnutrition

10. A mother brings her 2-month-old daughter to the pediatrician because she

feels a lump in her abdomen. On pelvic ultrasound the infant has an ovarian
mass. When counseling the mother, you inform her that the most common
ovarian lesion associated with the transient elevated gonadotropins in a
female newborn during the first 6 to 12 months of life is which of the
following?
(A) granulosa cell tumor
(B) leiomyoma
(C) serous cystadenoma
(D) single large follicular cyst
(E) theca cell tumor

Which of the following pubertal events is not mediated by


gonadal estrogen production and therefore would occur even in the
absence of estrogen production?
(A) breast development
(B) menstruation
(C) pubic hair growth
(D) accelerated skeletal growth
11.

(E) vaginal cornification

12. . A pregnant patient presents very concerned about some skin


lesions/changes she is seeing that are just like her uncles who has liver
cirrhosis from hepatitis C. What lesions or changes are she likely referring to?
(A) hyperpigmentation and spider angiomata
(B) linea nigra and chloasma
(C) spider angiomata and palmar erythema
(D) striae and chloasma
(E) striae and linea nigra

13. All the following are true about cardiac valve replacement EXCEPT:
A. Bioprosthetic valve replacement is preferred to mechanical valve
replacement in younger patients because of the superior durability of the
valve.
B. Bioprosthetic valves have a low incidence of thromboembolic
complications.
C. The risk of thrombosis with mechanical valve replacement is higher in the
mitral position than in the aortic position.
D. Mechanical valves are relatively contraindicated in patients who wish to
become pregnant.
E. Double-disk tilting mechanical prosthetic valves offer superior
hemodynamic characteristics over single-disk tilting valves.

14.

Labor is induced at 38 weeks due to severe oligo-hydramnios. The infant is born with a
congenital absence of the left hand. This is likely due to which of the following?

!(A) amniotic bands


(B) chorioangioma
(C) genetic abnormalities
(D) maternal trauma
(E) true knots in the umbilical cord !
15. Which of the following best describes the fetal kidneys?

(A) They are first capable of producing highly concentrated urine at 3 months.
(B) They are first capable of producing highly concentrated urine at 6 months.
(C) They are not affected by urinary tract obstruction in utero.
(D) If absent, they are associated with pulmonary hypoplasia.
(E) They produce only normotonic urine.

16. A poorly controlled class D diabetic patient desired a repeat cesarean


section. An amniocentesis to verify pulmonary maturity was done prior to
scheduled surgery at 37 weeks gestation. The L/S ratio was 2:1 and
phosphatidyl-glycerol was absent. An infant was delivered who developed
infant respiratory distress syndrome (IRDS). What was the most likely
reason?
(A) Diabetic patients do not produce lecithin.
(B) Fetal lung maturation may be delayed in maternal diabetes.
(C) Foam test was not done.
(D) The L/S test was done on fetal urine.
(E) Maternal blood was present in the specimen.
17. The smallest circumference of the normal fetal head corresponds to the

plane of which diameter?


(A) biparietal diameter
(B) bitemporal diameter
(C) occipitofrontal diameter
(D) occipitomental diameter
(E) suboccipitobregmatic diameter
18. A patient has an emergent cesarean section for an abruption. Because of
a large anterior placenta, the placenta was entered during the surgery. The
mother is Rh-negative. The infant appears anemic and is Rh-positive. To
determine the amount of RhoGAM that needs to be given to prevent
sensitization, an estimate of the amount of fetal red blood cells (RBCs) in the
maternal circulation is necessary. Fetal RBCs can be distinguished from
maternal RBCs by which of the following?

(A) lack of Rh factor


(B) lower amounts of hemoglobin
(C) nucleated RBCs
(D) resistance to acid elution
(E) shape
19. A woman presents for her new obstetrical visit at 12 weeks

EGA. Her medical history is complicated by Graves thyroiditis that


has been treated with radioactive iodine a few years prior. The
patient is currently being maintained on thyroid replacement. She
is worried that this will compromise the fetus. She is told that the
interaction between maternal and fetal physiology relative to
thyroid function is complex. Which of the following is an accurate
description of this interaction?
(A) Maternal thyroid hormones (T4 and T3) readily cross the
placenta.
(B) Maternal thyrotropin easily crosses the placenta.
(C) The athyroid fetus is growth retarded at birth.
(D) The fetal thyroid concentrates iodide.
(E) The placenta serves as a barrier to maternal iodine crossing to

the fetus.
20. A patient presents with a positive pregnancy test, the exact date of the
start of her last normal menses, and the date of her luteinizing hormone (LH)
surge from a urine kit. Her expected date of delivery can most correctly be
calculated by which of the following?
(A) adding 254 to the date of the start of the last menstrual period (LMP)
(B) counting 10 lunar months from the time of ovulation
(C) counting 280 from the first day of the LMP
(D) counting 40 weeks from the last day of the LMP
(E) adding 256 to the date of the elevated urinary LH when detected by home
testing
21. A patient develops a fever and tachycardia during a blood

transfusion after a redo coronary artery bypass procedure. The


nurse subsequently discovers that there was a mix-up in the crossmatch because of a labeling error. Which of the following is
diagnostic in a patient with an immediate hemolytic reaction
secondary to a blood transfusion?
a. Serum haptoglobin above 50 mg/dL
b. Indirect bilirubin greater than 5 mg/dL
c. Direct bilirubin greater than 5 mg/dL
d. Positive Coombs test
e. Myoglobinuria

22. Pregnancy should be avoided within 1 month of receiving which of the


following vaccinations?
(A) measles, mumps, rubella (MMR)
(B) influenza
(C) hepatitis B
(D) tetanus
(E) pneumococcus
23. During late pregnancy, which of the following implies urinary tract
disease?
(A) decreased serum creatinine
(B) failure to excrete concentrated urine after 18 hours without fluids
(C) glucosuria
(D) dilation of the ureters
(E) decreased creatinine clearance
24. You are seeing for the first prenatal visit a 19-year-old woman

with an artificial porcine valve placed 6 months ago for CHD. She
is 10 weeks pregnant, tired, and does not sleep particularly well.
The fetus is size-date appropriate. Of the following choices, which
is the best first step in management?
(A) evaluate for valve replacement due to assumed cardiac
enlargement
(B) anticoagulate with aspirin and a platelet inhibitor
(C) recommend termination of the pregnancy as she had not
postponed conception for the mandatory24 months after valve
replacement
(D) anticoagulate with heparin
(E) keep on low-dose oral antibiotics

25. A patient with suspected cholestasis of pregnancy develops a slight


hyperbilirubinemia and slight elevation of SGOT. You obtain serum bile salts that are
positive, confirming the diagnosis. Relief of the pruritus may be obtained by which of
the following?
(A) amitriptyline
(B) bland diet
(C) oral H2 blockers
(D) cholestyramine
(E) mild diuretic therapy
14.2.
1. A 10-year-old boy is brought to the psychiatrist by his mother. She states

that for the past 2 months he has been increasingly irritable, withdrawn, and
apathetic. He has been refusing to do his homework, and his grades have
dropped. Which of the following is the best next step in management?
a. The child should be hospitalized.
b. The child should be started in supportive psychotherapy.
c. The mother should be warned that the child will likely turn out to be bipolar

(67% chance).
d. The child should receive an antidepressant medication.
e. The child should receive lithium and an antidepressant.

2. A 32-year-old man is being treated for a severe major depression. Which of the
following symptoms, if present, is one of the most accurate indicators of long-term
suicidal risk?
a. Revenge fantasies
b. Presence of rage in the patient
c. Hopelessness
d. Presence of guilte. Presence of a need for punishment in patient

3. A 30-year-old man comes to the psychiatrist for the evaluation of a depressed mood. He
states that at least since his mid-20s he has felt depressed. He notes poor self-esteem and low
energy, and feels hopeless about his situation, though he denies suicidal ideation. He states he
does not use drugs or alcohol, and has no medical problems. His last physical examination by
his physician 1 month ago was entirely normal. Which of the following treatment options
should be tried first?
a. ECT
!b. Hospitalization
c. Psychoanalysis
d. Venlafaxine!
e. Amoxapine
This patient has a dysthymic disorder.

4. A 26-year-old man comes to the physician with the chief complaint of a


depressed mood for the past 5 weeks. He has been feeling down, with
decreased concentration, energy, and interest in his usual hobbies. Six weeks
prior to this office visit, he had been to the emergency room for an acute
asthma attack and was started on prednisone. Which of the following is the
most likely diagnosis?
a. Mood disorder secondary to a general medical condition
b. Substance-induced mood disorder
c. Major depression
d. Adjustment disorder
e. Dysthymia

5. A 56-year-old previously healthy physician notices that his eyes

are yellow and he has been losing weight. On physical


examination the patient has jaundice and scleral icterus with a
benign abdomen. Transcutaneous ultrasound of the abdomen
demonstrates biliary ductal dilation without gallstones.
Which of the following is the most appropriate next step in the
workup of this patient?
a. Esophagogastroduodenoscopy (EGD)
b. Endoscopic retrograde cholangiopancreatography (ERCP)
c. Acute abdominal series
d. Computed tomography (CT) scan
e. Positron emission tomography (PET) scan
15.2
1. banana and lemon signs are ultrasound findings in which of the following :
(A) interventricular cardiac defect
(B) NTD - Neural tube defects
(C) duodenal atresia
(D) Potters syndrome
(E) thalassemia

2. A 32-year-old woman is brought to the emergency room when she complains of


chest pain. She is noted to be hypervigilant and anxious, with a pulse of 120
beats/minute and BP of 140/97 mm/Hg. Her temperature is not elevated. She has
widely dilated pupils. Her toxicology screen is positive. Which of the following drugs
is she most likely to have used?
a. Cocaine
b. Ritalin
c. Heroin
d. Phencyclidine (PCP)
e. Lysergic acid diethylamide (LSD)

A succussion splash describes the sound obtained by shaking an individual who has free flu
gas in a hollow organ or body cavity.
It is usually elicited to confirm intestinal or pyloric obstruction due to pyloric stenosis or gas
carcinoma, but may also be heard with hydropneumothorax, a large hiatal hernia, or over a
stomach.
To examine for this sign, gently shake the abdomen by holding either side of the pelvis. A p
occurs when a splashing noise is heard, either with the naked ear, or with the aid of a steth
test is not valid if the patient has eaten or drunk fluid within the last three hours.

3. A 52-year-old woman undergoes a sigmoid resection with

primary anastomosis for recurrent diverticulitis. She returns to the


emergency room 10 days later with left flank pain and decreased
urine output; laboratory examination is significant for a white blood
cell (WBC) count of 20,000/mm3. She undergoes a CT scan that
demonstrates new left hydronephrosis, but no evidence of an intraabdominal abscess. Which of the following is the most appropriate
next step in management?
a. Intravenous pyelogram
b. b. Intravenous antibiotics and repeat CT in 1 week
c. Administration of intravenous methylene blue
d. No further management if urinalysis is negative for
hematuria
e. Immediate reexploration
4. Which of the following should be prescribed for the average
pregnant woman?
(A) an increase of no more than 1520 lb in pregnancy
(B) iron supplementation, in addition to prenatal vitamins
(C) continuation of moderate exercise
(D) 4-mg folic acid supplementation
(E) vinegar-water douches in the third trimester
5. An agitated patient is seen during the first trimester of
pregnancy with an enlarged thyroid, a BP of 110/70 mm Hg, a
resting pulse of 110, and an increased RBC uptake of
triiodothyronine T3 What should be your first step?
(A) measure thyroid-stimulating hormone (TSH)
(B) obtain an iodine 131 (I131) uptake by the thyroid
(C) obtain a basal metabolic rate (BMR)
(D) evaluate free thyroxine (T4)
(E) evaluate thyroid-binding globulin
TSH , Thyreoglobuline TOtal T3 and T4 may be flase increased in pregnancy
therefore they are not indicative for thyroid disorder in pregnancy
6. A cardiologist is called to consult on the care of a 2-day-old girl

delivered at 33 weeks gesta- tion. The infant is lying supine in her


isolette. She is acyanotic, but has a heart rate of 192/min and a
respiratory rate of 60/min. She has a nonradiating continuous
machinery mur- mur at the left upper sternal border that re- mains

the same with compression of the ipsilateral, then contralateral


jugular veins. Her first and second heart sounds are normal. Her
peripheral pulses are bounding. What is the most likely diagnosis?
(A) Aortic stenosis with aortic regurgitation
(B) Patent ductus arteriosus
(C) Systemic arteriovenous fistula
(D) Venous hum
(E) Ventricular septal defect
7.

. A 62-year-old woman undergoes a pancreaticoduodenectomy for a pancreatic head


cancer. A jejunostomy is placed to facilitate nutritional repletion as she is expected to have a
prolonged recovery. What is the best method for delivering postoperative nutrition?
a. Institution of enteral feeding via the jejunostomy tube after return of bowel function as
evidenced by
passage of flatus or a bowel movement
b. Institution of enteral feeding via the jejunostomy tube within 24 hours postoperatively
c. Institution of supplemental enteral feeding via the jejunostomy tube only if oral intake is
inadequate after return of bowel function
d. Institution of a combination of immediate trophic (15 mL/h) enteral feeds via the
jejunostomy tube and parenteral nutrition to provide total nutritional support
e. Complete nutritional support with total parenteral nutrition

8. A 26-year-old man is resuscitated with packed red blood cells


following a motor vehicle collision complicated by a fractured pelvis
and resultant hemorrhage. A few hours later the patient becomes
hypotensive with a normal central venous pressure (CVP), oliguric,
and febrile. Upon examination, the patient is noted to have profuse
oozing of blood from his intravenous (IV) sites. Which of the
following is the most likely diagnosis?
a. Hypovolemic shock
b. Acute adrenal insufficiency
c. Gram-negative bacteremia
d. Transfusion reaction
e. Ureteral obstruction
9. A 59-year-old man is planning to undergo a coronary artery
bypass. He has osteoarthritis and consumes nonsteroidal antiinflammatory drugs (NSAIDs) for the pain. Which of the following is
the most appropriate treatment prior to surgery to minimize his risk
of bleeding from his NSAID use?

a. Begin vitamin K 1 week prior to surgery.


b. Give FFP few hours before surgery.
c. Stop the NSAIDs 1 week prior to surgery.
d. Stop the NSAIDs 3 to 4 days prior to surgery.
e. Stop the NSAIDs the day before surgery.
10. Best managed medicallly without surgery:
A. A 74-year-old male with a dissection involving the root of the
aorta.
B. A 45-year-old female with a dissection involving the aorta distal
to the great vessel origin but cephalad to the renal arteries.
C. A 58-year-old male with aortic dissection involving the distal
aorta and the bilateral renal arteries.
D. A 69-year-old male with an intramural hematoma within the
aortic root.
E. All of the above patients require surgical manage- ment of their
aortic disease.
11. A 13-year-old female complains of heavy menses. Menarche
occurred 3 months ago. Her periods are irregular and consist of
bleeding for 5 to 7 days. She denies passing any blood clots. She
has to change her pad once during the night and 4 to 5 times
during the day. There are no bleeding disorders in her family. Her
hemoglobin is 10.. What is the most likely etiology of her
menometrorrhagia?
A. Pregnancy
B. Idiopathic thrombocytopenic purpura (ITP)
C. Cervical polyp
D. Foreign body
E. Immature hypothalamic-pituitary-ovarian axis
The majority of dysfunctional uterine bleeding in adolescents is due to
anovulatory cycles from an immature hypothalamic-pituitary-ovarian axis; 50%
to 75% of their cycles are anovulatory from menarche to 2 years after
menarche.
12. A 53-year-old woman has been intubated for several days after

sustaining a right pulmonary contusion after a motor vehicle


collision as well as multiple rib fractures. Which of the following is a
reasonable indication to attempt extubation?
a. Negative inspiratory force (NIF) of 15 cm H2O
b. PO2 of 60 mm Hg while breathing 30% inspired FiO2 with a

positive end-expiratory pressure (PEEP) of 10 cm H2O


c. Spontaneous respiratory rate of 35 breaths per minute
d. A rapid shallow breathing index of 80
e. Minute ventilation of 18 L/min
The rapid shallow breathing index is the ratio of the respiratory rate to tidal
volume. There is evidence to suggest that an index between 60 and 105
predicts successful extubation.

13. A 61-year-old alcoholic man presents with severe epigastric


pain radiating to his back. His amylase and lipase are elevated,
and he is diagnosed with acute pancreatitis. Over the first 48
hours, he is determined to have 6 Ransons criteria, including a
PaO2 less than 60 mm Hg. His chest x-ray reveals
bilateral pulmonary infiltrates, and his wedge pressure is low.
Which of the following criteria must be met to make a diagnosis of
adult respiratory distress syndrome (ARDS)?
a. Hypoxemia defined as a PaO2/FiO2 ratio of less than 200
b. Hypoxemia defined as a PaO2 of less than 60 mm Hg
c. A pulmonary capillary wedge pressure greater than 18 mm
Hg
d. Lack of improvement in oxygenation with administration of a
test dose of furosemide
e. Presence of a focal infiltrate on chest x-ray
The diagnosis can be made based on bilateral pulmonary infiltrates on chest x-ray, a
PaO /FiO ratio of less than 200, and pulmonary wedge pressures of less than 18 mm Hg
2
2

A 63-year-old man presents to the clinic for assessment of


muscle weakness symptoms. He also
mentions intermittent episodes of hemoptysis, but no fever, cough,
or sputum production. He hasa 60-pack-per-year history of
smoking. On physical examination, there is a blue purple
discoloration of the upper eyelids and erythema on his knuckles.
He also has proximal muscle
weakness rated 4+/5, normal reflexes, and sensation. A chest xray (CXR) reveals a lung masswith mediastinal widening. Which of
the following is the most likely diagnosis for his muscleweakness?
14.

(A) SLE
(B) scleroderma

(C) dermatomyositis (DM)


(D) polyarteritis
(E) WeberChristian disease

15. A 32-year-old man undergoes a distal pancreatectomy,


splenectomy, and partial colectomy for a gunshot wound to the left
upper quadrant of the abdomen. One week later he develops a
shaking chill in conjunction with a temperature spike of 39.4C
(103F). His blood pressure is 70/40 mm Hg, pulse is 140 beats
per minute, and respiratory rate is 45 breaths per minute. He is
transferred to the intensive care unit (ICU), where he is intubated
and a Swan-Ganz catheter is placed. Which of the following is
consistent with the expected initial Swan-Ganz catheter readings?
a. An increase in cardiac output
b. An increase in peripheral vascular resistance
c. An increase in pulmonary artery pressure
d. An increase in PCWP
e. An increase in central venous pressure
The case presented is most consistent with septic shock from a postoperative intra-abdominal
abscess. In the early phase of septic shock, the respiratory profile is characterized by mild
hypoxia with a compensatory hyperventilation and respiratory alkalosis. Hemodynamically, a
hyperdynamic state is seen with an increase in cardiac output and a decrease in peripheral
vascular resistance in the face of relatively normal central pressures.

16. A

55-year-old woman requires an abdominoperineal operation


for rectal cancer. She has a history of stable angina. Which of the
following clinical markers is most likely to predict a cardiac event
during her noncardiac surgery and should prompt further cardiac
workup prior to her operation?
a. Abnormal electrocardiogram
b. Prior stroke
c. Unstable angina
d. Uncontrolled hypertension

e. Her age
Major predictors include unstable angina, recent myocardial infarction,
decompensated congestive heart failure, significant arrhythmias, and severe valvular
disease.

17. Which of the following is the most common organism

seen in spontaneous bacterial peritonitis?


a. P. aeruginosa
b. Enterococcus
c. S. pneumoniae
d. Enterobacteriaceae
e. Anaerobes

18. A 22-year-old man undergoes an exploratory laparotomy after


a gunshot wound to the abdomen. The patient has multiple
injuries, including a significant liver laceration, multiple small-bowel
and colon injuries, and an injury to the infrahepatic vena cava. The
patient receives 35 units of packed RBCs, 15 L of crystalloid, 12
units of fresh-frozen plasma (FFP), and a 12 packs of platelets.
The patients abdomen is packed closed and he is taken to the
intensive care unit for further resuscitation. Which of the following
warrants a decompressive laparotomy?
a. Increased peak airway pressure
b. Increased cardiac output
c. Decreased systemic vascular resistance
d. Decreased plasma renin and aldosterone
e. Increased cerebral perfusion pressure
There are multiple sequelae of increased abdominal pressure or abdominal compartment
syndrome. Compartment syndrome results in increased peak airway pressures, decreased
venous return and decreased cardiac output, increased systemic vascular resistance, decreased
renal blood flow and glomerular filtration rate, and decreased portal venous flow with
decreased liver function. Because of decreased venous return, the intracranial pressure
increases and cerebral perfusion pressure decreases. Treatment requires decompressive
laparotomy.

A 21-year-old man is diagnosed with post- streptococcal


glomerulonephritis. Which of the following is likely to be found in
his urine?
A. Greater than 3 g/24-hour proteinuria without hematuria
B. Macroscopic hematuria and 24-hour urinary albumin
of 227 mg
C. Microscopic hematuria with leukocytes and 24-hoururinary
albumin of 227 mg
19.

D. Positive urine culture for Streptococcus


E. Sterile pyuria without proteinuria

20. A 58-year-old woman undergoes a hysterectomy and


postoperatively develops acute respiratory distress syndrome. She
is treated with mechanical ventilation and broad-spectrum
antibiotics. Aside from hypothyroidism, she has no underlying
medical conditions. On day 5 of her hospitalization her urine output
is noted to fall and her serum creatinine rises from 1.2 mg/dL to
2.5 mg/dL. Allergic interstitial nephritis from cephalosporin
antibiotics is suspected. Which of the following findings will confirm
this diagnosis?
A. Hematuria
B. Peripheral blood eosinophilia
C. Urinary eosinophils on urine microscopy
D. White blood cell casts on urine microscopy
E. None of the above

21. It is hospital day 5 for a 65-year-old patient with prerenal


azotemia secondary to dehydration. His creatinine was initially 3.6
mg/dL on admission, but it has improved today to 2.1 mg/dL. He
complains of mild lower back pain, and you prescribe naproxen to
be taken intermittently. By what mechanism might this drug further
impair his renal function?
A. Afferent arteriolar vasoconstriction

B. Afferent arteriolar vasodilatation


C. Efferent arteriolar vasoconstriction
D. Proximal tubular toxicity
E. Ureteral obstruction
22. A 42-year-old man is undergoing chemotherapy after resection
of a cecal adenocarcinoma with positive lymph nodes. You are
asked to see him regarding a potential surgical complication.
Which of the following potentially operable complications is a
common occurrence among patients receiving systemic
chemotherapy?
a. Acute cholecystitis
b. Perirectal abscess
c. Appendicitis
d. Incarcerated femoral hernia
e. Diverticulitis
23.

A 28-year-old medical student seeks your attention because of a testicular mass. Biopsy
is consistent with pure seminoma. There is no evidence of enlarged retroperitoneal lymph
nodes on CT scan. Which of the following is the best treatment strategy for this patient?
a. Orchiectomy alone
b. Orchiectomy followed by chemotherapy
c. Orchiectomy with retroperitoneal lymph node dissection
d. Orchiectomy with retroperitoneal lymph node dissection followed by external beam
radiation
e. Orchiectomy followed by external beam radiation to the retroperitoneal lymph nodes

In evaluation for acute kidney injury in a patient who has


recently undergone cardiopulmonary bypass during mitral valve
replacement, which of the following findings on urine microscopy is
most suggestive of cholesterol emboli as the source of renal
failure?
A. Calcium oxalate crystals
B. Eosinophiluria
C. Granular casts
D. Normal sediment
24.

E. White blood cell casts

25. A 22-year-old woman has a known family history of breast


cancer in her first-degree relatives. She undergoes genetic testing
and is found to be a BRCA1 mutation carrier. She does not
currently desire bilateral prophylactic mastectomy. Which of the
following is the next best option to manage her risk for breast
cancer?
a. Mammography every 6 months starting at age 25
b. Mammography every 6 months starting at age 35
c. Mammography every 12 months starting at age 25
d. Mammography every 12 months starting at age 35
e. Tamoxifen for chemoprevention

26.

A 52-year-old woman sees her physician with complaints of fatigue, headache, flank
pain, hematuria, and abdominal pain. She undergoes a sestamibi scan that demonstrates
persistent uptake in the right superior parathyroid gland at 2 hours. Which of the following
laboratory values is most suggestive of her diagnosis?
a. Serum acid phosphatase above 120 IU/L
b. Serum alkaline phosphatase above 120 IU/L
c. Serum calcium above 11 mg/dL
d. Urinary calcium below 100 mg/day
e. Parathyroid hormone levels below 5 pmol/L

A 55-year-old woman presents with a 6-cm right thyroid mass


and palpable cervical lymphadenopathy. Fine-needle aspiration
(FNA) of one of the lymph nodes demonstrates the presence of
calcified clumps of sloughed cells. Which of the following best
describes the management of this thyroid disorder?
a. The patient should be screened for pancreatic endocrine
neoplasms and hypercalcemia.
b. The patient should undergo total thyroidectomy with modified
radical neck dissection.
c. The patient should undergo total thyroidectomy with frozen
section intraoperatively, with modified radical neck dissection
reserved for patients with extra-capsular invasion.
27.

d. The patient should undergo right thyroid lobectomy followed by


iodine 131 (131I) therapy.
e. The patient should undergo right thyroid lobectomy.
26. A 30-year-old woman presents with hypertension, weakness, bone pain, and a serum
calcium level of 15.2 mg/dL. Hand films below show osteitis fibrosa cystica. Which of the
following is the most likely cause of these findings?
a. Sarcoidosis
b. Vitamin D intoxication
c. Paget disease
d. Metastatic carcinoma
e. Primary hyperparathyroidism
Osteitis fibrosa cystica is a condition associated with hyperparathyroidism that is
characterized by severe demineralization with subperiosteal bone resorption

27 !A!38!year!old!woman!visits!her!family!physician!because!of!her!concern!about!a!

lump!in!her!left!breast!which!she!noticed!after!a!fall!approximately!one!week!
previously.!She!doesnt!smoke,!drinks!wine!on!social!occasions,!and!takes!birth!
control!pills.!There!is!no!family!history!of!breast!cancer.!Physical!examination!reveals!
a!rather!stocky!women!with!pendulous!breasts.!She!has!a!moderated!sized,!
nontender,!irregular!lump!in!the!lower!outer!quadrant!of!her!left!breast.!The!
overlying!skin!appears!thickened.!No!nipple!discharge!is!present,!and!she!has!no!
axillary!lymphadenopathy.!The!right!breast!and!axilla!are!normal.!Mammography!
reveals!increased!density!in!the!affected!region.!Ultrasound!guided!fine!needle!
aspiration!(FNAB)!was!performed.!The!pathology!report!was!consistent!with!a!
diagnoses!of!fat!necrosis.!Which!of!the!following!is!the!most!appropriate!next!step!in!
the!management!of!this!patient?!
a.!Excise!the!mass.!!
b.!Repeat!fine!needle!aspiration!biopsy!after!one!month.!!
c.!Repeat!mammography!in!a!month.!!
d.!Reassure!the!patient!and!follow!up!in!a!few!weeks.!!

Fat necrosis is a benign (not cancer) breast condition

A 14-year-old female has progressive headaches.Examination


shows bilateral papillary edema.CT-scan of the brain is
normal.What should be the next diagnostic test?
A.lumber puncture with opening pressure
B.MRI of the brain
C.orbital CT-scan
D.urine toxicology screen
E.serum beta HCG measurement
28.

29.!A!24!year!old!male!playing!football!fell!on!his!outstretched!hand!hoping!to!make!
a!touchdown.!The!patient!complains!on!severe!pain!in!his!right!arm.!His!vital!signs!
are:!BP!140/80,!PR!98!regular,!Temp!37C!and!RR!22/min!regular.!The!right!arm!is!
swollen!and!angulated!in!the!mid!arm!area,!it's!tender!to!touch!and!movement!is!
painful.!The!humeral!appear!to!be!fractured,!capillary!circulation!in!the!nail!bed!is!
normal.!However,!there!is!a!clinical!evidence!of!nerve!damage.!X[ray!imaging!
confirms!fracture!of!the!humeral!shaft!with!some!angulated!displacement.!Which!of!
the!following!is!the!nerve!injury!associated!with!this!fracture:!
a.!Axiliary!nerve!!
b.!Median!nerve!
!c.!Ulnar!nerve!!
d.!Radial!nerve!!
e.!Brachial!nerve!Correct!

A 65-year-old woman went bicycling into the country near Lake


Tahoe, California, with a few of her friends. She had not bicycled
since her childhood growing up at the foothills of the Ural
Mountains in Russia. The front wheel struck a pothole and she fell
to the ground. While doing so, her chest struck the handlebar. She
was immediately rushed to the local hospital. On arrival, she is
conscious and appears to be in moderate distress. Her blood
pressure is 140/ 95 mm Hg, pulse 96/min. Her respirations are
shallow and rapid, at 22/min. Bruising over the left anterior
hemithorax is observed upon examination of the chest. There is no
jugular venous dilatation. The rest of her examination findings are
unremarkable. Further examination would most likely yield which of
30.

the following?
(A) Increased breath sounds on the left
(B) Egophony on the left
(C) Increased timbre
(D) Increased width of intercostals on the left
(E) Mediastinal structures shifted to the left
This patient has sustained a pneumothorax in the left hemithorax secondary to
trauma. A pneumothorax can occur spontaneously or secondary to trauma
resulting from a blunt or penetrating injury. A pneumothorax is the presence of
air trapped in the pleural space that is unavailable for gaseous exchange.
Physical findings include hyperresonance to percussion (also known increased
timbre) on the side of the problem (choice C).

31.!While riding his bicycle, a 9-year-old boy loses control and falls.
During the process, his abdomen strikes the handlebar. His
parents bring him to the emergency department because he has
vague midabdominal pain and some bruising of the
anteriorabdominal wall. His vital signs are stable, and he has no
other visible injuries. Which of the following is the most likely
diagnosis?
(A) Ruptured spleen
(B) Ruptured liver
(C) Ruptured pancreas
(D) Hematoma in the rectus muscle
(E) Ruptured duodenum

The presentation of this patient is classic for rupture of the duodenum (choice
E). Patients have vague symptoms because the duodenum is retroperitoneal. If
left untreated, mortality is almost 100%. The best way to diagnose it is to
maintain a high index of suspicion and conduct repeated physical
examinations. Serum amylase is often elevated but is not diagnostic. X-ray
films of the abdomen reveal retroperitoneal air, which is the sine qua non of
duodenal rupture. Duodenal rupture in children could also result from the use
of the lap belt without shoulder support in motor vehicles. This results from
acute hyperflexion of the thoracolumbar spine, which crushes the duodenum.
Drivers of motor vehicles can also sustain duodenal rupture as a result of compression against the steering wheel. This possibility is greatly reduced by an air
bag.

32. A 73-year-old man with Scandinavian ancestors was undergoing a routine


physical examination when his physician noted a roundish spot about 6 mm (14
inch) in diameter that was slightly pinker than the remainder of the mans face.
When he ran his finger across it, he also noted that was dry and rough. The
patient informed him that something had been going on with that spot for a
year or so; its appearance seemed to wax and wane, essentially disappear- ing
once in a while but then returning, sometimes covered with white scales that
then dropped off. When asked, the patient admitted that as a boy and a young
man he spent countless hours at the beach body surfing and never used
sunscreen. The physician cauterized the lesion with liquid nitrogen. Within 2
weeks the area became crusted, shrank, and fell off. Two weeks thereafter, the
rough spot returned, and after a month, the patient developed an open sore at
the same location that would not heal. The sore is most likely to be which one
of the following?
(A) Actinic keratosis!
(B) Basal cell carcinoma!
(C) Squamous cell carcinoma
(D) Nodular melanoma!
(E) Herpes infection

33. Buspirone (Buspar) acts as a

A. dopamine partial agonist useful in the treatment of OCD


B. serotonin partial agonist useful in the treatment of OCD
C. dopamine partial agonist useful in the treatment of generalized
anxiety disorder
D. serotonin partial agonist useful in treatment of generalized
anxiety disorder
E. none of the above

34. A 52-year-old woman is brought to the emergency room after her husband finds her
unresponsive at home. The patient left behind a suicide note, and two empty bottles of pills
(sertraline and lorazepam) plus an empty bottle of vodka were found next to the patient. In the
emergency room the patients vital signs are: blood pressure 90/60 mm Hg, pulse 60
beats/minute, respirations 6 breaths/minute. Which of the following medications is most
likely to be helpful in the emergency

room setting in this situation?


a. Acamprosate
b. Zolpidem!
c. Flumazenil
d. LAAM
e. Disulfiram

This patients respiration is very depressed, and it


is likely that the benzodiazepines are playing a role.
Flumazenil can be used in the emergency room
setting to counteract the effects of benzodiazepine
overdose.
35. A 32-year-old man comes to the physician with complaints of insomnia. He states
for the past 3 weeks he has had difficulty going to sleep, though once he finally gets
to sleep, he stays asleep without difficulty. The patient states that he is having no
other difficulties. The patient has a past history of alcohol dependence, though he
has been sober for over 3 years. Which of the following medications is the best
choice to prescribe to help the patient with his sleep?

a. Ramelteon
b. Trazodone
c. Zolpidem
d. Triazolam
e. Zaleplon

Ramelteon mimics melatonins sleep-inducing properties. It has a high affinity for


melatonin MT1 and MT2 receptors in the brain. The half-life of ramelteon is between
1 and 2.5 hours. Ramelteon reduces time to sleep onset, and to a lesser extent,
increases the amount of time spent in sleep. The most common side effect is
headache. It should not be used in patients with severe hepatic impairment, severe
sleep apnea, or severe COPD. There has been no evidence found of rebound insomnia
or withdrawal effects from this drug.

36. Generalized anxiety disorder :

A. is least likely to coexist with another mental disorder


B. has a female-to-male ratio of 1:2
C. is a mild condition
D. has about a 50 percent chance of a recurrence after recovery
E. has a low prevalence in primary care settings

37. A 19-year-old man presents to the clinic for evaluation of low

back pain. The discomfort is dull in quality and associated with


morning stiffness. He reports no fever, chills, night sweats, or
weight loss. His past medical history is significant for a chronic
papulosquamous skin disorder involving his knees and elbows. On
physical examination, forward flexion at the lumbar spine
is reduced but neck movements are normal. There is stress
tenderness of both sacroiliac joints.On serologic testing the RF
and ANA are negative. Which of the following is the most
likelydiagnosis?
(A) RA
(B) ankylosing spondylitis
(C) psoriatic arthritis
(D) reactive arthritis
(E) PMR
38. A 27-year-old G4P3 at 37 weeks presents to the hospital with

heavy vaginal bleeding and painful uterine contractions. Quick


bedside ultrasound reveals a fundal placenta. The patients vital
signs are blood pressure 140/92 mm Hg, pulse 118 beats per
minute, respiratory rate 20 breaths per minute, and temperature
37C (98.6F). The fetal heart rate tracing reveals tachycardia with
decreased variability and a few late decelerations. An emergency
cesarean section delivers a male infant with Apgar scores of 4 and
9. With delivery of the placenta, a large retroplacental clot is noted.
The patient becomes hypotensive, and bleeding is noted from the
wound edges and her IV catheter sites. Which of the following

blood products will most quickly resolve her cause of hemorrhage?


a. Cryoprecipitate
b. Fresh frozen plasma
c. Packed red blood cells
d. Platelets
e. Recombinant Factor VII
This patient has a large placental abruption which is the most common
cause of consumptive coagulopathy in pregnancy. The bleeding described
signifies that the patient has a significant coagulopathy with
hypofibrinogenemia. Prompt and vigorous transfusion is needed. Packed
red blood cells will restore blood volume and increase oxygen carrying
capacity. Fresh frozen plasma (FFP) contains about 600-700 mg of
fibrinogen and will promote clotting. Cryoprecipitate contains clotting
factors and fibrinogen but in much less amount (200 mg) than FFP and
has no advantage over the use of FFP in this bleeding patient.

39. A 30-year-old man complains of unilateral headaches. He was


diagnosed with migraine headaches at age 24. The headaches did
not respond to triptan therapy at that time, but after 6 weeks the
headaches resolved. He has had three or four spells of severe
headaches since then. Currently his headaches have been present
for the past 2 weeks. The headaches start with a stabbing pain just
below the right eye.Usually the affected eye feels irritated
(reddened with increased lacrimation). He saw an
optometristduring one of the episodes and a miotic pupil was
noted. Each pain lasts from 60 to 90 minutes, but hemay have
several discrete episodes each day. The neurological examination,
including cranial nerveexamination, is now normal. What is your
best approach to treatment at this time?
a. Prescribe oral sumatriptan for use at the onset of headache.
b. Prednisone 60 mg daily for 2 to 4 weeks.
c. Obtain MRI scan of the head with gadolinium contrast.
d. Begin propranolol 20 mg bid.
e. Refer for neuropsychiatric testing
Treatment of cluster headache involves two principles: (1) aborting the cluster
and (2) relieving the
headache when it occurs. Prednisone is usually given to abort the cluster; 40
to 60 mg per day is given for
weeks and then tapered over a month or two.

40. A 26-year-old G1P1 is now postoperative day (POD) 6 after a

low transverse cesarean delivery for arrest of active phase. On


POD 2, the patient developed a fever of 39C (102.2F) and was
noted to have uterine tenderness and foul-smelling lochia. She
was started on broad-spectrum antibiotic coverage for
endometritis. The patient states she feels fine now and wants to go
home, but continues to spike fevers each evening. Her lung,
breast, and cardiac examinations are normal. Her abdomen is
nontender with firm, nontender uterus below the umbilicus. On
pelvic examination her uterus is appropriately enlarged, but
nontender. The adnexa are nontender without masses. Her lochia
is normal. Her white blood cell count is 12 with a normal
differential. Blood, sputum, and urine cultures are all negative for
growth after 3 days. Her chest x-ray is negative. Which of the
following statements is true regarding this patients condition?
a. It usually involves both the iliofemoral and ovarian veins.
b. Antimicrobial therapy is usually ineffective.
c.Fever spikes are rare.
d. Heparin therapy is always needed for resolution of fever.
e. Vena caval thrombosis may accompany either ovarian or
iliofemoral thrombophlebitis.
There are two types of septic pelvic thrombophlebitis (SPT): ovarian
vein thrombophlebitis (OVT) and deep septic pelvic thrombophlebitis
(DSPT). These two entities share common pathogenic mechanisms and
often occur together, but they may differ in their clinical presentations
and diagnostic findings.Patients with OVT usually present with fever
and abdominal pain within one week after delivery or surgery, and
thrombosis of the right ovarian vein is visualized radiographically in
about 20 percent of cases. Patients with DSPT usually present within a
few days after delivery or surgery with unlocalized fever that persists
despite antibiotics, in the absence of radiographic evidence of
thrombosis.

41. A 21-year-old G1 at 40 weeks, who underwent induction of

labor for severe preeclampsia, delivered a 3900-g male infant via

vaginal delivery after pushing for 212 hours. A second-degree


midline laceration and side-wall laceration were repaired in the
usual fashion under local analgesia. The estimated blood loss was
450 cc. Magnesium sulfate is continued postpartum for the seizure
prophylaxis. Six hours after the delivery, the patient has difficulty
voiding. A distended bladder can be palpated suprapubically and
bladder catherization produces 1000 cc of urine. Which is the
following statement regarding postpartum urinary retention is true?
a. Bladder catherization in labor increases its risk.
b. It is more common in multiparous patients.
c. Labor epidural reduces the risk of urinary retention.
d. Occurs less commonly with instrumented deliveries.
e. Most commonly occurs after short labors.
42. A 53-year-old male patient is found to have an occipital lobe
tumor. He would be least likely to exhibit which of the following
symptoms and complaints?
A. Headache
B. Homonymous hemianopsia
C. Papilledema
D. Paranoid delusions
E. Visual hallucinations
43. A 30-year-old G5P3 has undergone a repeat cesarean
delivery. She wants to breast-feed. Her past medical history is
significant for hepatitis B infection, hypothyroidism, depression,
and breast reduction. She is receiving intravenous antibiotics for
endometritis. Which of the following would prevent her from breastfeeding?
a. Maternal reduction mammoplasty with transplantation of the
nipples
b. Maternal treatment with ampicillin
c. Maternal treatment with fluoxetine
d. Maternal treatment with levothyroxine
e. Past hepatitis B infection
44. Which of the following stressors most often leads to
psychological impairment that could be diagnosed as an
adjustment disorder?
A. Loss of a job
B. A plane crash

C. Rape
D. All of the above
E. None of the above
45. A 23-year-old G2P1 develops chorioamnionitis during labor
and is started on ampicillin and gentamicin. She requires a
cesarean delivery for arrest of active phase labor and the same
antibiotics are continued after surgery. On postoperative day 3, the
patient remains febrile and symptomatic with uterine fundal
tenderness. No masses are appreciated by pelvic examination.
She is successfully breast-feeding and her breast examination is
normal. Which antibiotic should be initiated to provide better
coverage?
a. Cefazolin
b. Clindamycin
c. Moxifloxacin
d. Piperacillin with tazobactam
e. Vancomycin
46. A 23-year-old G2P1 develops chorioamnionitis during labor
and is started on ampicillin and gentamicin. She requires a
cesarean delivery for arrest of active phase labor and the same
antibiotics are continued after surgery. On postoperative day 3, the
patient remains febrile and symptomatic with uterine fundal
tenderness. No masses are appreciated by pelvic examination.
She is successfully breast-feeding and her breast examination is
normal. Which antibiotic should be initiated to provide better
coverage?
a. Cefazolin
b. Clindamycin
c. Moxifloxacin
d. Piperacillin with tazobactam
e. Vancomycin
Following vaginal delivery the antibiotic treatment of choice is ampicillin and
gentamicin, the same as the treatment for chorioamnionitis. After cesarean
delivery, broad- spectrum coverage with clindamycin and gentamicin is the
standard of care, and ampicillin is added to cover enterococcus if fever
persists after 48 to 72 hours. Clindamycin should be used for the treatment of
infections after cesarean delivery to provide anaerobic coverage.

47. You are asked to assist in the well-born nursery with neonatal care. Which of the
following is a part of routine care in a healthy infant?!
a. Administration of ceftriaxone cream to the eyes for prophylaxis for gonorrhea and
chlamydia!
b. Administration of vitamin A to prevent bleeding problems
c. Administration of hepatitis B vaccination for routine immunization!
d. Cool-water bath to remove vernix!
e. Placement of a computer chip in left buttock for identification purposes

Following a vaginal delivery, a woman develops fever, lower


abdominal pain, and uterine tenderness. She is alert, and her
blood pressure and urine output are good. She started on broadspectrum antibiotics, but continues to spike fevers 4 days
postpartum. Which of the following is most closely tied to a
decision to proceed with hysterectomy?
a. Desires sterilization
b. Imaging showing pelvic abscess
c. Imaging showing gas in the myometrium
d. Persistent fever greater than 102F
48.

e. Septic shock

49. A 19-year-old primiparous woman develops postpartum hemorrhage


unresponsive to oxytocin and uterine massage. Her infant was 8.5 pounds. She has
bled 750 cc. What is the most likely diagnosis?
(A) laceration(s) of cervix or vagina
(B) placenta accreta
(C) uterine inversion
(D) ruptured uterus
(E) coagulopathy

50. Certain patients are more likely than others to have uterine

atony and hemorrhage after delivery. Circumstances that predict


possible increased bleeding postpartum include which of the
following situations?
(A) prolonged labor
(B) primigravidas
(C) hypertensive disorders
(D) pudendal anesthesia for delivery
(E) obesity

51.

A 42-year-old G4P2012 has had a complicated prenatal course due to chronic


hypertension and Type 2 diabetes. She also has adult onset asthma made worse with aspirin.
Because of worsening hypertension an induction was initiated. She dilated to 5 cm and then
did not progress further over the next 3 hours. Labor dystocia is likely due to which of the
following issues?
(A) maternal hypertensive disorders
(B) ineffective uterine contractions
(C) maternal pulmonary disease
(D) hyperglycemia

52. You are checking a 22-year-old primigravida in active labor. Labor has lasted 14

hours. She is 8-cm dilated and at 0 station. As the fetal head has descended, the shape
has changed. Which of the following is the most likely etiology?
(A) cephalohematoma
(B) molding
(C) subdural hematomas
(D) hydrocephalus
(E) caput succedaneum

53. Epidural anesthesia is preferable in which maternal condition?

(A) to avoid maternal valsalva with bearing down in mother with


previous cesarean section
(B) multiple gestation
(C) maternal cardiac disease
(D) compound presentation
E) premature labor with a fetus <28 weeks
54. Which of the following situations would be likely to have the
complication of a contracted pelvis?
(A) Marfans disease
(B) long-standing maternal drug use
(C) pendulous abdomen in a primigravida
(D) morbidly obese multigravida
(E) short maternal stature

55.

Which of the following indications most likely predict a classic cesarean section as
opposed to a traditional transverse lower uterine segment cesarean section?
(A) maternal Crohns disease
(B) vertical skin incision, unknown previous uterine incision
(C) 26-week gestation with a breech presentation
(D) fundal myoma
(E) twins, with the first baby in a breech presentation

53.!Term labors lasting less than 3 hours are associated with which of the following
conditions?
(A) decreased fetal morbidity
(B) less maternal morbidity
(C) increased fetal morbidity
(D) primiparous labors
(E) twins

16.2
You are a passenger aboard an airplane and a 78-year-old
woman is complaining of chest pain and difficulty breathing. You
are the only medical professional available and volunteer to help.
Fortunately, the aircraft is well-equipped with basic medical
equipment, as well as with ACLS medications and a cardiac
monitor. On examination, the passengers BP is 75/40 mm Hg, HR
is 180 beats per minute, and RR is 24 breaths per minute. On
examination, the patient is in obvious distress but able to answer
basic questions. Her heart is tachycardic, regular, and without
murmurs, rubs, or gallops. Physical examination is remarkable for
a bounding carotid pulse. You attach the cardiac monitor and see a
regular rhythm at 180 beats per minute with wide QRS complexes
and no obvious P waves. After asking the pilot to make an
emergency landing, what do you do next?
a. Amiodarone IV
b. Synchronized cardioversion
c. Verapamil IV
d. Lidocaine IV
e. Procainamide IV
1.

2. A 41-year-old man is brought into the ED by paramedics in cardiopulmonary


arrest. A friend states that the patient is a long-time user of IV heroin. You look at the
monitor and see that the patient has pulseless electrical activity (PEA).
Cardiopulmonary resuscitation is being performed and the patient is intubated. You
decide to administer epinephrine to the patient but realize that he does not have IV
access. Which of the following drugs is ineffective when administered through an
endotracheal (ET) tube?

a. Atropine
a. Atropine!
b. Naloxone!
c. Lidocaine!
d. Epinephrine!
e. Sodium bicarbonate

Classically, six dysfunctional labor patterns are defined, including


(1) prolonged latent phase,
(2) protracted active phase dilation,
(3) secondary arrest of dilation,
(4) prolonged deceleration phase, - If engagement has not occurred
by 9 cm of dilation, the likelihood of a second-stage abnormality is
increased. Prolonged deceleration phase is strongly associated with
descent disorders. Typically, prolonged deceleration phase as a
labor abnormality seems most closely related to secondary arrest.
Careful augmentation of labor with oxytocin, as per the protocol
described earlier, is often warranted.
(5) protracted descent, and
(6) arrest of descent

A 30-year-old G2P0 at 39 weeks is admitted in active labor with


spontaneous rupture of membranes occurring 2 hours prior to
admission. The patient noted clear fluid at the time. On
examination, her cervix is 4 cm dilated and completely effaced.
The fetal head is at 0 station and the fetal heart rate tracing is
reactive. Two hours later on repeat examination her cervix is 5 cm
dilated and the fetal head is at +1 station. Early decelerations are
noted on the fetal heart rate tracing. Which of the following is the
best next step in her labor management?
3.

a. Administer terbutaline.
b. Initiate amnioinfusion.
c. Initiate Pitocin augmentation.
d. Perform cesarean delivery for arrest of descent.
e. Perform cesarean delivery of early decelerations.

4. A 2-year-old boy has been vomiting intermittently for 3 weeks


and hasbeen irritable, listless, and anorectic. His use of language
has regressed to speaking
single words. In your evaluation of this patient, which of the
following is the most reasonable diagnosis to consider?
a. Expanding epidural hematoma
b. Herpes simplex virus (HSV) encephalitis
c. Tuberculous meningitis
d. Food allergy
e. Bacterial meningitis

5. A pregnant woman at 32 weeks is brought to the emergency


department after a motor vehicle accident with abdominal trauma.
The fetus is dead and the mother is in shock. You diagnose an
abruption and go to the operating room with trauma surgeons for a
possible C-section delivery. You find bleeding into the myometrium
beneath the uterine serosa. In severe cases, what is the cause of
abruptio placentae?
(A) uteroplacental apoplexy
(B) uterine rupture
(C) minimal effect on fetal heart rate
(D) adnexal torsion
(E) disseminated intravascular coagulopathy (DIC)
bleeding into the myometrium is called uteroplacental apoplexy ( couvelier
uterus)

6. Eight minutes after a normal delivery under pudendal anesthesia, the patient has
not completed the third stage of labor. The uterus is discoid and firm; no bleeding is
evident. What should you do?
(A) pull steadily but with greater traction on the cord
(B) perform Credes maneuver
(C) augment the contractions with intramuscular
(IM) methergine
(D) manually remove the placenta
(E) gently massage the uterus and wait

7.A 21-year-old primiparous patient arrives in Labor and Delivery


with poor prenatal care, her last visit being 8 weeks ago. She is 41
weeks by dates, and states she ruptured membranes
approximately 12 hours ago. On examination, estimated weight is
9 lb. She has thick meconium. Her cervix is 3 cm, dilated, and the
presenting part is at 2 station. The presenting part is a face. The
fetal heart tones are excellent; she is not contracting. What is the
best management?
(A) oxytocin induction with group B strep prophylaxis
(B) Misoprostol 25 g, group B strep prophylaxis
(C) expectant management with group B strep prophylaxis
(D) epidural anesthesia to relax the pelvis, group B strep
prophylaxis, and then oxytocin
(E) cesarean section
8. A 32-year-old woman (gravida 3, para 1, abortus 1) at term is admitted in
labor with an initial cervical examination of 6-cm dilatation, complete
effacement, and the vertex at 1 station. Estimated fetal weight is 8 lb, and
her first pregnancy resulted in an uncomplicated vaginal delivery of an 8-lb
infant. After 2 hours, there is no cervical change. An intrauterine pressure
catheter is placed. This shows three contractions in a 10-minute period, each
with a strength of 40 mm Hg.. What is this abnormality of labor termed?
(A) prolonged latent phase
(B) active-phase arrest
(C) failure of descent
(D) arrest of latent phase
(E) protraction of descent
No cervical change after 2 hours is defined as an active- phase arrest. Arrest is a term used in
situations where no change in a process has been noted over a period of time (commonly in
12 hours)

A couple comes in to see you because they have been


unsuccessfully trying to conceive for the past 14 months. After a
thorough history, you learn that the woman has never been
pregnant and that the man has fathered no children to his
knowledge. You also learn that the timing of their intercourse has
been appropriate for conception. As an initial workup, what do you
do next?
a. Semen analysis for the husband and menstrual tracking,
ovulation tracking, and thyroid-stimulating hormone, folliclestimulating hormone, and prolactin levels for the wife
9.

b. Semen analysis and testicular biopsy for the husband and


ovulation tracking, hysterosalpingogram, and laparoscopy for the
wife
c. Offer the couple clomiphene citrate (Clomid) because it has few
side effects and the best success rate in couples with unexplained
infertility
d. Refer the couple to an infertility specialist capable of performing
in vitro fertilization
e. Do nothing because this couple is not technically infertile until
they have been trying to conceive

10. 3. A 47-year-old G3 P3 black female comes in to see you for a


second opinion. She has had fibroids all of her life and has always
had regularly timed menses but they were very heavy. She has
never bled after intercourse or between her periods. She has
become accustomed to these symptoms but is concerned because
her last period lasted 10 days and she was changing her
menstrual pad nearly every 2 to 3 hours. Her TSH and endometrial
biopsy were within normal limits. Her hematocrit at the visit is 31%.
A repeat pelvic U/S shows a multifibroid uterus that is 18 cm in
overall size. Three years ago her uterus was 15 cm in size and 4
years before that it was 13 cm in size. Her primary gynecologist
has suggested that she may need a hysterectomy and she wants
your opinion. In this situation, the patient's symptoms would best
be treated with:
a. oral contraceptive pills.
b. total abdominal hysterectomy.
c. depo-lupron.
d.hysteroscopic myomectomy.
e. abdominal myomectomy.

11. Term labors lasting less than 3 hours are associated with
which of the following conditions?
(A) decreased fetal morbidity
(B) less maternal morbidity
(C) increased fetal morbidity
(D) primiparous labors
(E) twins

12. A 28-year-old woman presents to her internist with a 2-day


history of low-grade fever and lower abdominal pain. She denies
nausea, vomiting, or diarrhea. On physical
examination, there is temperature of 38.3C (100.9F) and bilateral
lower quadrant tenderness, without point or rebound tenderness.
Bowel sounds are normal. On pelvic examination, an exudate is
present and there is tenderness on motion of the cervix. Her white
blood cell count is 15,000/L and urinalysis shows no red or white
blood cells.Serum -hCG is undetectable. Which of the following is
the best next step in management?
a. T reatment with ceftriaxone and doxycycline
b.T reatment with ciprofloxacin and metronidazole
c. Diagnostic laparoscopy
d. CT scan of abdomen and pelvis
e. Aztreonam
T his patient presents with the clinical picture of pelvic inflammatory disease
(PID), including lower quadrant tenderness, cervical motion tenderness, and
adnexal tenderness. Fever and mucopurulent discharge are additional
evidence for the diagnosis. T reatment requires antibiotic therapy. Ceftriaxone
and doxycycline are one recommended regimen
13. Examination of an asymptomatic 2-day-old infant girl shows a

distended abdomen. The urinary bladder and rectal ampulla are


empty. A solitary unilocular cyst is visualized with ultrasonography.
Which of the following is the best next step in the management of
this patient?
(A) observation
(B) intravenous pyelogram (IVP)
(C) cystoscopy
(D) barium enema(E) exploratory surgery

14. A 6-year-old girl has a history of 2 weeks of abdominal pain. She is significantly
taller than her peers. Physical examination shows early breast development and
abdominal distention. Blood is present at the introitus, and pelvic examination is
attempted but cannot be accomplished. Serum gonadotropin levels are in the
prepubertal range and do not change after gonadotropin-releasing hormone (GnRH)
administration. Abdominal sonography shows a 6-cm solid right adnexal mass. Which
of the following is the most likely diagnosis?
!(A) epophoron
(B) granulosa cell tumor
(C) corpus luteum cyst !
(D) endometrioma
(E) fibroma

A 1-year-old girl has an abdominal mass. Rectal examination


demonstrates a mass extending into the right pelvis. The cervix is
not palpable. Abdominal sonography shows that the uterus and
vagina are absent. Both ovaries appear normal. Which of the
following is the most likely origin of the mass?
(A) gastrointestinal
(B) renal
(C) musculoskeletal
(D) hepatic
(E) pancreatic
15.

16. A 32-year-old G2P2 develops fever and uterine tenderness 1


day after cesarean delivery for nonreassuring fetal heart tones.
She started on broad-spectrum antibiotics. On postoperative day
4, the patient remains febrile and symptomatic with abdominal and
pelvic tenderness. Her lung and cardiovascular exams are normal
and she has no costovertebral angle tenderness. She is
successfully breast-feeding and her breast examination is normal.
Computed tomography (CT) of the abdomen shows a small
bladder flap hematoma. What is the next best step in the
management of this patient?
a. CT-directed needle drainage of hematoma
b. Continued treatment with antibiotics
c. Exploratory laparotomy and drainage of hematoma
d. Hysterectomy

17. A 28-year-old G2P2 presents to the hospital 2 weeks after vaginal delivery with
the complaint of heavy vaginal bleeding that soaks a sanitary napkin every hour. Her
pulse is 89 beats per minute, blood pressure 120/76 mm Hg, and temperature 37.1C
(98.9F). Her abdomen is nontender and her fundus is located above the symphysis
pubis. On pelvic examination, her vagina contained small blood clots and no active
bleeding is noted from the cervix. Her uterus is about 12 to 14 weeks size and
nontender. Her cervix is closed. An ultrasound reveals an 8-mm endometrial stripe.
Her hemoglobin is 10.9, unchanged from the one at her vaginal delivery. -hCG is
negative. Which of the following potential treatments would be contraindicated?
a. Methylergonovine maleate (Methergine)
b. Oxytocin injection (Pitocin)!
c. Ergonovine maleate (Ergotrate)!
d. Prostaglandins
e. Dilation and curettage

18. 5. You are assisting in a gynecologic oncology clinic when you

see a 57-year-old G3 P3 female patient who is a former nurse. She


presents with 6 months of pelvic discomfort, increasing abdominal
girth, and early satiety. Physical exam reveals a large
abdominopelvic mass. A pelvic ultrasound and CT scan show a
10-cm right ovarian mass, ascites, and studding of the peritoneum.
In your discussion with the patient you predict that this most likely
represents a malignant ovarian neoplasm. She asks about the
primary method of treatment for ovarian carcinoma. You explain
that the mainstay of treatment for epithelial ovarian cancer is:

a. radiation therapy alone.

b. surgery alone.
c. surgery followed by chemotherapy.
d. surgery followed by radiation therapy.
d. chemoradiation alone.
19. A 50-year-old woman is diagnosed with cervical cancer. Which

lymph node group would be the first involved in metastatic spread


of this disease beyond the cervix and uterus?
a. Common iliac nodes
b. Sacral nodes
c. External iliac nodes
d. Paracervical nodes
e. Para-aortic nodes

20. Your next patient is returning to see you to get her pelvic

ultrasound results. She is 32-year-old G2 P1011 who presents with


menorrhagia for the past year. She denies any intermenstrual
bleeding or postcoital spotting. However, when her menses come,
she needs to wear an overnight pad and a super tampon at the
same time. For the first 3 days, she has to change her pad and
tampon about every 2 hours. She's already ruined several pairs of
pants and has had to leave work twice for heavy bleeding. Her
pelvic ultrasound is notable for an 8-cm uterus with two intramural
fibroids about 2 cm each. She has a third fibroid that is 3.1 cm.
About one-half of that fibroid is reported as being submucosal. Her
ovaries are both normal. Her TSH from last visit was normal, her
hematocrit is 33%, and her endometrial biopsy showed
proliferative endometrium. She wants your advice for what to do
next. How would this patient be best managed?
a. Intravenous estrogen
b. Oral contraceptive pills
c. Hysteroscopic myomectomy
d. Abdominal myomectomy
e. Abdominal hysterectomy

21. A 19-year-old primiparous woman develops toxemia in her last


trimester of pregnancy and during the course of her labor is treated
with magnesium sulfate. At 38 weeks gestation, she delivers a
2100-g (4-lb, 10-oz) infant with
Apgar scores of 1 at 1 minute and 5 at 5 minutes. Laboratory
studies at 18 hours of age reveal a hematocrit of 79%, platelet
count of 100,000/L,glucose 41 mg/dL, magnesium 2.5 mEq/L,
and calcium 8.7 mg/dL. Soonafter, the infant has a generalized
convulsion. Which of the following is themost likely cause of the
infants seizure?
a. Polycythemia
b. Hypoglycemia
c. Hypocalcemia
d. Hypermagnesemia
e. Thrombocytopenia
A persistent venous hematocrit of
greater than 65% in a neonate is regarded as polycythemia and will be
accompanied
by an increase in blood viscosity

22. Which of the following aneuploidies would be most common in the setting of otherwise
normal screening?
Trisomy 21 !
Trisomy 18 !
Trisomy 13 !
Sex chromosomal aneuploidy !
None of the above !
23.

A28-year-old woman comes to

the emergency department complaining of 1 day of worsening right


leg pain and swelling. She drove in a car for 8 hours returning from
a hiking trip 2 days ago then noticed some pain in the leg. At first
she thought it was due to exertion but it has worsened over the
day. Her only past medical history is related to difficulty getting
pregnant with 2 prior spontaneous abortions. Her physical
examination is notable for normal vital signs and heart and lung

examination. Her right leg is swollen from the mid-thigh down and
is tender. Doppler studies demonstrate a large deep venous
thrombosis in the femoral and ileac veins extending into the pelvis.
Laboratory studies on admission prior to therapy show normal
electrolytes, normal white blood cell (WBC) and platelet counts,
normal prothrombin time, and an activated partial thromboplastin
time 3 normal. Her pregnancy test is negative. Low- molecularweight heparin therapy is initiated in the emergency department.
Subsequent therapy should include:
A. Rituximab 375 mg/m2 per week for 4 weeks
B. Warfarin with INR goal of 2.03.0 for 3 months
C. Warfarin with INR goal of 2.03.0 for 12 months
D. Warfarin with INR goal of 2.53.5 for life
E. Warfarin with an INR goal of 2.53.5 for 12 monthsfollowed by
daily aspirin for life
Anti phospholipid syndrome : After diagnosis of a thrombotic event due to APS,
patients should receive warfarin for life with a goal INR of 2.53.5 alone or in combination
with daily aspirin.

24.!All the following are pulmonary manifestations of systemic lupus


erythematosus EXCEPT:
A. Cavitary lung nodules
B.Diaphragmatic dysfunction with loss of lung volumes
C.Pleuritis
D.Pulmonary hemorrhage
E.Pulmonary vascular disease

25. Your patient presents for her first prenatal visit. She is 27-year-

old and this is her first pregnancy. She is an achondroplastic


dwarf. Her husband is of normal stature. Which of the following
statements should you tell her regarding achondroplasia?
a. The inheritance pattern is autosomal recessive therefore there is
a one-in-four chance that her child
will be affected.
b. Achondroplasia is caused by a new genetic mutation therefore it

cannot be passed on to her child.


c. Because she has achondroplasia she has a low risk of cesarean
section for delivery.
d. She is fortunate to have lived to reproductive age.
e. She likely has some degree of spinal stenosis which could
present a difficulty with spinal orepidural anesthesia.
26. A 44-year-old pregnant woman is trying to choose chorionic
villi sampling (CVS) versus amniocentesis for prenatal diagnosis
due to her increased risk of having a child with a chromosomal
anomaly. Which of the following is an advantage of amniocentesis
over CVS?
a. Amniocentesis can be performed earlier in pregnancy than CVS.
b. Amniocentesis in any trimester is less painful than CVS.
c. A second-trimester diagnosis of an abnormal karyotype afforded
by amniocentesis allows for safertermination of pregnancy if
termination is chosen by the patient.
d. Mid-trimester amniocentesis has a lower complication rate than
CVS.
e. First-trimester amniocentesis has a lower complication rate than
CVS.
27. Following an uncomplicated delivery, cyanosis develops in a
3.7-kg term infant during the first hour of life. The findings at 3
hours of age include cyanosis, heart rate of 140 beats/minute,
respiratory rate of 56 breaths/minute, and no heart murmurs. The
pulse oximetry reading in room air is 70% in the right hand and
75% in the foot; with a 100% fraction of inspired oxygen (FIO2) by
head hood, the oxygen saturation remains 70% in the hand but
increases to 90% in the foot. The chest roentgenographic findings
are normal.These findings are most consistent with:
a. Primary pulmonary hypertension of the newborn
b. Pulmonary valve atresia
c. Transient tachypnea of the newborn
d. Transposition of the great arteries
e. Truncus arteriosus

28. A 65-year-old with a history of chronic atrial fibrillation is being


monitored while on warfarin
therapy.The nurse calls to inform you the patient's International
Normalized Ratio (INR) is measured
at 7. He has no active signs of bleeding, but is at increased risk of
bleeding. Appropriate management
at this time includes
A) stop warfarin, observe, and repeat INR in 3 days
B) stop warfarin and observe; repeat INR in 24 hours
C) stop warfarin, give vitamin K, and repeat INR in 24 hours
D) stop warfarin, give vitamin K and fresh frozen plasma with daily
INRs
29. A patient is brought to the ER after a motor vehicle accident.

He is unconscious and has a deep scalp laceration and one dilated


pupil. His heart rate is 120 beats per minute blood pressure is
80/40 mm/Hg, and respiratory rate 35 per min. Despite rapid
administration of 2 L normal saline, and patients vital signs do not
change significantly. Which of the following is the most appropriate
next step in the
workup of his hypotension?
a. Neurosurgical consultation for emergent ventriculostomy to
manage his intracranial pressure
b. Neurosurgical consulation for emergent craniotoymy for
suspected subdural hematoma
c. Emergent burr hole draining at the bedside for suspected
epidural hematoma
d. Administration of mannitol and hyperventilation to treat his
elevated intracranial pressure
e. Abdominal ultrasound (focused assessment with sonography in
trauma, FAST)

25. A 37-year-old G7 P6 with a dichorionic/diamnionic,

vertex/vertex twin gestation at 38 weeks presents to labor and


delivery for induction of labor. She is started on oxytocin and
begins having contractions after several hours. The patient
progresses slowly over the next 16 hours until she is 5 cm dilated
and, at this point, develops a fever and fetal tachycardia. The
woman is diagnosed with chorioamnionitis and antibiotic therapy is
started. She delivers the babies vaginally 6 hours later. Right after
delivery of the second infant, there is a large, continuous
hemorrhage from the vagina. The most likely cause of this is:
a. vaginal laceration.
b. cervical laceration.
c. uterine atony.
d. uterine rupture.
e. placenta accreta.

30. A 3-year-old girl has a mild febrile illness with mild URI
symptoms. She has an erythematous rash on both cheeks. Her
pregnant mother had arthralgias of the hands wrists, knees, and
ankles a week ago. What should be the next action?
(
A) Closely monitor the childs sibling who has spherocytosis.
(B) Exclude the child from daycare until rash has resolved.
(C) Exclude the pregnant daycare providers until no further cases
are diagnosed for 2 weeks.
(D) Give the mother IVIG.
(E) Give the sibling with spherocytosis IVIG.

31. Two weeks after the birth of her infant, a new mother brings the
child in to see you. The childs eyes are edematous, with
conjunctival erythema and a mucopurulent discharge. Your
evaluation and treatment should include which of the following?
(A) a pelvic examination (using a small scope) of the infant
(B) culture maternal genital tract for GC and chlamydia
(C) anaerobic cultures of the infants and mothers eyes
(D) immunoglobulin M (IgM) titers of the infant
(E) penicillin VK for both the mother and the infant

32. A 43-year-old woman has had a history of frequency, urgency,


and dysuria for the past 8 years. She has had five negative urine
cultures and urinalyses in the last year. Cystoscopy 1 month ago
showed a normal bladder and reddened urethra. An intravenous
pyelogram (IVP) is normal. What is the most likely diagnosis?
(A) surreptitious use of antibiotics by the patient to mask her
laboratory results
(B) tuberculous urethritis
(C) vulvar vestibulitis syndrome
(D) urethral syndrome
(E) urethral gonorrhea
Urethral syndrome has same symptoms as cystitis. Its etiology is unknown, and the only
physical findings may be tenderness and redness of the urethra on urethroscopic examination.

33. A 51-year-old woman presents complaining of dysuria,


dyspareunia, frequency of urination, dribbling of urine from the
urethra when she stands after voiding, and a painful swelling under
her urethra. Which of the following is the most likely diagnosis?
(A) simple cystitis
(B) urethral syndrome
(C) infection of the Skenes glands
(D) infected urethral diverticulum
(E) urethral carcinoma
34. A 9-year-old girl presents for evaluation of regular vaginal

bleeding. History reveals thelarche at age 7 and adrenarche at age


8. Which of the following is the most common cause of this
condition in girls?
a. Idiopathic
b. Gonadal tumors
c. McCune-Albright syndrome
d. Hypothyroidism
e. Tumors of the central nervous system

35. A mother brings her 14-year-old daughter in to the office for


consultation. The mother says her daughter should have started
her period by now. She is also concerned that she is shorter than
her friends. On physical examination, the girl is 4 ft 10 in tall. She
shows evidence of breast development at Tanner stage 2. She has
no axillary or pubic hair. You reassure the mother that her
daughter seems to be developing normally. Educating the mother
and daughter, your best advice is to tell them which of the
following?
a. The daughter will start her period when her breasts reach
Tanner stage 5.
b. The daughter will start her period, then have her growth spurt.
c. The daughters period should start within 1 to 2 years since she
has just started developing breast buds.
d. The daughter will have her growth spurt, then pubic hair will
develop, heralding the onset ofmenstruation.
e. The daughters period should start by age 18, but if she has not
had her period by then, she shouldcome back in for further
evaluation.

36. A 56-year-old man is hospitalized under the care of a vascular


surgeon for claudication and severe peripheral arterial disease.
You are asked to see the patient as a medical consultant to assess
periopera-tive risk for planned femoral-popliteal bypass graft
surgery. The patient reports a history of hypertension, smoking,
diabetes mellitus type 1, and a stroke 5 years ago. He denies a
history of heart or kidney disease. His medications include aspirin,
clopidogrel, amlodipine, benazepril, and insulin. He reports inability
to walk long distances due to calf pain, but he is able to perform
household chores, including doing laundry and dish washing,
without discomfort. He denies chest discomfort, dyspnea, or
edema. Blood pressure is 150/80 mm Hg, and pulse is 80/min.
Examination is normal except for a cool left leg with absent pedal
pulses. Serum creatinine and glucose levels are normal.
Electrocardiogram reveals normal sinus rhythm without
abnormality.
Which of the following is the most appropriate recommendation?
A. Proceed to surgery without further testing and consider adding
atenolol and simvastatin.
B. Proceed to surgery without further testing and discontinue
insulin when the patient is NPO.
C. Request a cardiac stress test before operating.
D. Discontinue aspirin and clopidogrel and wait 7-10 days before
operating.
E. Abandon surgical intervention, as the patients perioperative risk
is too high to justify the procedure

37. A 19-year-old G1P0 presents to her obstetricians office for a


routine OB visit at 32 weeks gestation. Her pregnancy has been
complicated by gestational diabetes requiring insulin for control.
She has been noncompliant with diet and insulin therapy. She has
had two prior normal ultrasound examinations at 20 and 28 weeks
gestation. She has no other significant past medical or surgical
history. During the visit, her fundal height measures 38 cm. Which
of the following is the most likely explanation for the discrepancy
between the fundal height and the gestational age?
a. Fetal hydrocephaly
b. Uterine fibroids
c. Polyhydramnios
d. Breech presentation
e. Undiagnosed twin gestation

Uterine fibroids, polyhydramnios (excessive


amniotic fluid), fetal macrosomia, and twin
gestation are all plausible explanations of why the
uterine size would measure larger than expected for
the patients dates.

38. A 43-year-old G1P0 who conceived via in vitro fertilization


comes into the office for her routine OB visit at 38 weeks. She
denies any problems since she was seen the week before. She
reports good fetal movement and denies any leakage of fluid per
vagina, vaginal bleeding, or regular uterine contractions. She
reports that sometimes she feels crampy at the end of the day
when she gets home from work, but this discomfort is alleviated
with getting off her feet. The fundal height measurement is 36 cm;
it measured 37 cm the week before. Her cervical examination is 2
cm dilated and the fetal head is engaged. Which of the following is
the most appropriate next step in the management of this patient?
a. Instruct the patient to return to the office in 1 week for her next
routine visit.
b. Admit the patient for induction of labor for a diagnosis of fetal
growth lag.
c. Send the patient for a sonogram to determine the amniotic fluid
index.
d. Order the patient to undergo a nonstress test.e. Do a fern test in
the office.
The decrease in fundal height between visits can be explained by engagement of the fetal
head, which is verified on vaginal examination with determination of the presenting part at 0
station. Engagement of the fetal head commonly occurs before labor in nulliparous patients.
Therefore it is appropriate for the patient to return for another scheduled visit in a week.

39. A 30-year-old G2P1001 patient comes to see you in the office


at 37 weeks gestational age for her routine OB visit. Her first
pregnancy resulted in a vaginal delivery of a 9-lb 8-oz baby boy
after 30 minutes of pushing. On doing Leopold maneuvers during
this office visit, you determine that the fetus is breech. Vaginal
examination demonstrates that the cervix is 50% effaced and 1 to
2 cm dilated. The presenting breech is high out of the pelvis. The
estimated fetal weight is about 7 lb. The patient denies having any
contractions. You send the patient for a sonogram, which confirms
a fetus with a double footling breech presentation. There is a
normal amount of amniotic fluid present and the head is
hyperextended in the stargazer position. Which of the following is
the best next step in the management of this patient?
a. Allow the patient to undergo a vaginal breech delivery whenever
she goes into labor.
b. Send the patient to labor and delivery immediately for an
emergent cesarean section.
c. Schedule a cesarean section at or after 41 weeks gestational
age.
d. Schedule an external cephalic version in the next few days.
e. Allow the patient to go into labor and do an external cephalic
version at that time if the fetus is still in the double footling breech
presentation.
40. A 29-year-old G1P0 presents to the obstetricians office at 41

weeks gestation. On physical examination, her cervix is 1


centimeter dilated, 0% effaced, firm, and posterior in position. The
vertex is presenting at 3 station. Which of the following is the best
next step in the management of this patient?
a. Send the patient to the hospital for induction of labor since she
has a favorable Bishop score.
b. Teach the patient to measure fetal kick counts and deliver her if
at any time there are less than 20perceived fetal movements in 3
hours.
c. Order biophysical profile testing (BPP) for the same or next day.
d. Schedule the patient for induction of labor at 43 weeks
gestation.
e. Schedule cesarean delivery for the following day since it is
unlikely that the patient will go intolabor.

41. Your patient had an ultrasound examination today at 39 weeks


gestation for size less than dates. The ultrasound showed
oligohydramnios with an amniotic fluid index of 1.5 centimeters.
The patients cervix is unfavorable. Which of the following is the
best next step in the management of this patient?
a. Admit her to the hospital for cesarean delivery.
b. Admit her to the hospital for cervical ripening then induction of
labor.
c. Write her a prescription for misoprostol to take at home orally
every 4 hours until she goes intolabor.d. Perform stripping of the
fetal membranes and perform a BPP in 2 days.
e. Administer a cervical ripening agent in your office and have the
patient present to the hospital inthe morning for induction with
oxytocin.
41. A 27-year-old gravida you have been following throughout her pregnancy
presents at 22 weeks gestation not feeling well. She complains of fever,
cough, a runny nose, conjunctivitis, and on examination has white spots
surrounded by a halo of erythema on her buccal mucosa and an
erythematous maculopapular rash on her abdomen. What is the most likely
cause of this patients condition?
(A) varicella zoster
(B) rubella
(C) rubeola (measles)
(D) syphilis
(E) herpes
(C) This is a classic presentation of measles (rubeola), especially the pathognomonic
(Kop-lik) spots. It seldom has an effect in midgesta-tion. Vaccination is not indicated
during pregnancy, and if you already have the infection, vaccination is of no value.
Some authorities would recommend passive immunity.

42. The hospital is reviewing its protocols to decrease the iatrogenic infection
rate within the hospital. For which of the following procedures would
prophylactic antibiotics be appropriate?
(A) amniocentesis
(B) laparoscopy
(C) tubal sterilization
(D) vaginal hysterectomy
(E) episiotomy repair
43. Regarding immunization during pregnancy, which of the following
vaccines would be the safest to receive during pregnancy?
(A) mumps
(B) polio
(C) rabies
(D) rubella
(E) rubeola (measles)

44. Engagement occurs when the biparietal diameter of the fetal


head descends below the level of which of the following?
a. midpelvis
b. pelvic inlet
c. pelvic floor
d. ischial tuberosities
45. Which of the following is a side effect of using indomethacin for the
management of hydramnios?
a. altered neonatal bleeding times
b. increased amnionic fluid volume
c. constriction of the fetal ductus arteriosus
d. premature separation of the placenta
46. Which of the following is not indicated in the emergent

treatment of thyroid storm?


A) Propylthiouracil
B) Supersaturated potassium iodine
C) Propranolol
D) Aspirin
E) Acetaminophen

47. Which is not use for therapy of uterine myomas:


a. Uterine artery embolizatione
b. Myoma extraction
c. Use focused Ultra sound
d. Cryolysise.
Hysteroscopic excision

48. An 18-year-old woman presents for care because of condom


broke during sexual intercourse. Coitus occurred 1 day ago when
she was at midcycle. She does not wish to be pregnant and will
terminate the pregnancy if menses does not occur. Regarding her
fear of pregnancy.
which is the most appropriate next step in her management?
(A) advise her that unprotected midcycle coitus has 5% risk of
pregnancy
(B) prescribe intravaginal misoprostol (Cytotec)
(C) advise immediate douching
(D) prescribe a brief course of levonorgestrel
(E) advise her to await her next menses before taking any action

49. Which of the following statements about altitude sickness is


true?
A) Most people are affected at altitudes between 5,000 and 7,500
ft.
B) Dehydration is rarely an associated condition.
C) The most common symptom is headache.
D) Hydrochlorothiazide is used for prophylaxis
.E) A high carbohydrate diet can help prevent symptoms.
50. A 34-year-old woman (gravida 2, para 1) is at 13 weeks
gestation by last menstrual period (LMP) with a desired pregnancy.
She presents to the emergency department very anxious with a
10-hour history of low abdominal cramping and vaginal bleeding.
Her temperature is 102.2F, and her uterus is markedly tender on
bimanual examination. Ultrasound shows an intrauterine
pregnancy with a crown- rump length consistent with her LMP and

fetal cardiac activity present. Her cervix is dilated by 1 cm. Her


WBC count is 26,000. What is the best management for this
patient?
(A) place a cervical cerclage immediately after administering
antibiotics
(B) administer antibiotics and expectantly manage her
(C) evacuate her uterus after administering antibiotics
(D) administer antibiotics, and if she does not spontaneously abort
after 24 hours of observation,
place a cervical cerclage
(E) place her on bed rest and administer both a tocolytic and
antibiotics

50. A 31-year-old infertility patient with regular ovulatory menstrual


cycles has begun therapy with clomiphene citrate. Before she
starts therapy.what information should you provide her regarding
the medication?
(A) Typically, the timing of ovulation is increased by a week.
(B) Approximately 40% of patients will respond to clomiphene
citrate with increased endometrial thickness.
(C) The risk of multiple gestation is 25%.
(D) Clomiphene citrate improves the fecundity rate principally
through its effect on the endometrial lining.
(E) Risk and side effects of clomiphene citrate include nausea, hot
flushes, weight gain, and mood swings.

Potrebbero piacerti anche